Health Ass 4: Endocrine Flashcards

1
Q

What is required to maintain a normal glucose level in the body?
A. The absorption of amino acids.
B. A balance between glucose usage and endogenous production or dietary carbohydrate intake.
C. The elimination of fats from the diet.
D. A constant supply of exogenous insulin.

A

Correct Answer: B. A balance between glucose usage and endogenous production or dietary carbohydrate intake.

How well did you know this?
1
Not at all
2
3
4
5
Perfectly
2
Q

Which organ is primarily responsible for endogenous glucose production?
A. The pancreas.
B. The spleen.
C. The liver.
D. The kidneys.

A

Correct Answer: C. The liver.

The liver is the primary source of endogenous glucose production via glycogenolysis & gluconeogenesis

How well did you know this?
1
Not at all
2
3
4
5
Perfectly
3
Q

After eating, when glucose usage outpaces production, what process occurs to stabilize blood glucose levels?
A. Conversion from glucose production to protein synthesis.
B. A transition from exogenous to endogenous glucose production.
C. The body increases insulin resistance.
D. Activation of lipid metabolism.

A

Correct Answer: B. A transition from exogenous to endogenous glucose production.

2-4 hours after eating, when glucose usage exceeds production, a transition from exogenous usage to endogenous production occurs to maintain a normal plasma glucose level

How well did you know this?
1
Not at all
2
3
4
5
Perfectly
4
Q

What percentage of glucose released by the liver is used by insulin-insensitive tissues?
A. 10-20%
B. 30-40%
C. 50-60%
D. 70-80%

A

Correct Answer: D. 70-80%.

70-80% of the glucose released by the liver is metabolized by insulin-insensitive tissues such as the brain, GI tract, and red blood cells

How well did you know this?
1
Not at all
2
3
4
5
Perfectly
5
Q

Which hormones are part of the glucose counterregulatory system?
A. Insulin and somatostatin.
B. Glucagon, epinephrine, growth hormone, and cortisol.
C. Thyroid hormones and leptin.
D. Estrogen and progesterone.

A

Correct Answer: B. Glucagon, epinephrine, growth hormone, and cortisol.

Rationale: These hormones act to increase blood glucose levels through various mechanisms, counteracting the glucose-lowering effects of insulin.

How well did you know this?
1
Not at all
2
3
4
5
Perfectly
6
Q

What role does glucagon play in glucose metabolism?
A. Decreases blood glucose levels by promoting glycogenesis.
B. Increases blood glucose levels by stimulating glycogenolysis and gluconeogenesis.
C. Has no effect on glucose metabolism.
D. Increases glucose uptake into the cells.

A

Correct Answer: B. Increases blood glucose levels by stimulating glycogenolysis and gluconeogenesis.

Rationale: Glucagon’s primary function is to raise blood glucose levels by breaking down glycogen into glucose (glycogenolysis) and creating new glucose (gluconeogenesis), particularly in the fasting state. It also inhibits the breakdown of glucose (glycolysis).

How well did you know this?
1
Not at all
2
3
4
5
Perfectly
7
Q

What is the prevalence of diabetes mellitus among adults?
A. 1 in 5 adults.
B. 1 in 10 adults.
C. 1 in 20 adults.
D. 1 in 50 adults.

A

Correct Answer: B. 1 in 10 adults.
Rationale: Diabetes mellitus affects 1 in 10 adults, making it the most common endocrine disease.

How well did you know this?
1
Not at all
2
3
4
5
Perfectly
8
Q

What is the etiology of Type 1a diabetes?
A. Genetic mutation leading to insulin overproduction.
B. Lifestyle factors leading to insulin resistance.
C. T-cell mediated autoimmune destruction of β cells.
D. Viral infection leading to pancreatic damage.

A

Correct Answer: C. T-cell mediated autoimmune destruction of β cells.

Rationale: Type 1a diabetes is caused by an autoimmune reaction where T-cells attack and destroy β cells in the pancreas, leading to minimal or absent insulin production.

How well did you know this?
1
Not at all
2
3
4
5
Perfectly
9
Q

How does Type 2 diabetes differ from Type 1 regarding its pathophysiology?
A. Type 2 is an autoimmune disease, while Type 1 is not.
B. Type 2 results from a complete deficiency of insulin.
C. Type 2 is characterized by defects in insulin signaling, not an autoimmune process.
D. Type 2 is a transient condition, while Type 1 is permanent.

A

Correct Answer: C. Type 2 is characterized by defects in insulin signaling, not an autoimmune process.

Rationale: Unlike Type 1 diabetes, which involves autoimmune destruction of β cells, Type 2 diabetes arises from defects in insulin receptors and subsequent intracellular signaling, leading to insulin resistance.

Type 1b diabetes is a rare disease of absolute insulin deficiency, which is not immune mediated

How well did you know this?
1
Not at all
2
3
4
5
Perfectly
10
Q

What percentage of diabetes mellitus (DM) cases does Type 1 diabetes comprise?
A. Less than 1%.
B. 5-10%.
C. 25-30%.
D. Over 50%.

A

Correct Answer: B. 5-10%.

Rationale: Type 1 diabetes accounts for 5-10% of all diabetes cases, distinguishing it as less common than Type 2 diabetes.

How well did you know this?
1
Not at all
2
3
4
5
Perfectly
11
Q

When is Type 1 diabetes typically diagnosed?
A. In infancy.
B. Before age 40.
C. After age 60.
D. It has no typical age of onset.

A

Correct Answer: B. Before age 40.

Rationale: Type 1 diabetes is often diagnosed in younger individuals, commonly before age 40.

How well did you know this?
1
Not at all
2
3
4
5
Perfectly
12
Q

What proportion of B cell function is typically lost before hyperglycemia becomes apparent in Type 1 diabetes?
A. Around 10-20%.
B. About 50-60%.
C. At least 80-90%.
D. 100%.

A

Correct Answer: C. At least 80-90%.

Rationale: In Type 1 diabetes, a significant loss of B cell function, generally 80-90%, occurs before the clinical manifestations of hyperglycemia are evident.

How well did you know this?
1
Not at all
2
3
4
5
Perfectly
13
Q

Which of the following is a symptom of hyperglycemia in Type 1 diabetes?
A. Hypoglycemia.
B. Weight gain.
C. Decreased urination.
D. Polyuria.

A

Correct Answer: D. Polyuria.

Rationale: Hyperglycemia can lead to polyuria (increased urination), polydipsia (increased thirst), fatigue, weight loss, and potentially, if left untreated, to more serious conditions like ketoacidosis.

How well did you know this?
1
Not at all
2
3
4
5
Perfectly
14
Q

What percentage of diabetes mellitus (DM) cases does Type 2 diabetes constitute?
A. Less than 10%.
B. Approximately 50%.
C. More than 90%.
D. Exactly 75%.

A

Correct Answer: C. More than 90%.

How well did you know this?
1
Not at all
2
3
4
5
Perfectly
15
Q

How long before diagnosis do individuals with Type 2 diabetes typically exhibit symptoms?
A. 1-3 years.
B. 4-7 years.
C. 8-10 years.
D. Over 10 years.

A

Correct Answer: B. 4-7 years.

under-diagnosed

How well did you know this?
1
Not at all
2
3
4
5
Perfectly
16
Q

Which of the following is a main abnormality seen in Type 2 diabetes?
A. Decreased hepatic glucose release.
B. Excess insulin secretion by the pancreas.
C. Increased glucose uptake in peripheral tissues.
D. Impaired insulin secretion.

A

Correct Answer: D. Impaired insulin secretion.

3 main abnormalities seen in DM2:
↑hepatic glucose release *c/b a reduction in insulin’s inhibitory effect on liver,
Impaired insulin secretion,
Insufficient glucose uptake in peripheral tissues

How well did you know this?
1
Not at all
2
3
4
5
Perfectly
17
Q

Type 2 diabetes is characterized by insulin insensitivity in peripheral tissues. What is the initial response of the pancreas to this insensitivity?
A. Decrease in insulin secretion.
B. Increase in glucagon secretion.
C. Increase in insulin secretion.
D. No change in insulin secretion.

A

Correct Answer: C. Increase in insulin secretion.

How well did you know this?
1
Not at all
2
3
4
5
Perfectly
18
Q

Which of the following molecular abnormalities does NOT directly contribute to insulin resistance in Type 2 Diabetes?
A) Deficient insulin secretion
B) Abnormal insulin molecules
C) Circulating insulin antagonists
D) Insulin receptor defects

A

Correct Answer: A) Deficient insulin secretion

Rationale: Insulin resistance is characterized by the body’s impaired response to insulin, not necessarily by a deficiency in insulin secretion, which is more characteristic of Type 1 Diabetes. The slide mentions abnormalities in insulin molecules, circulating antagonists, and receptor defects as contributing factors to insulin resistance, but not deficient secretion of insulin itself.

How well did you know this?
1
Not at all
2
3
4
5
Perfectly
19
Q

In the context of Type 2 Diabetes, how might an increased level of circulating insulin antagonists exacerbate the condition?
A) By increasing insulin sensitivity
B) By inhibiting insulin secretion
C) By decreasing insulin degradation
D) By competing with insulin for receptor binding

A

Correct Answer: D) By competing with insulin for receptor binding

Rationale: Circulating insulin antagonists would compete with insulin for binding to its receptor, thereby impeding insulin’s action and worsening insulin resistance. The other options are either beneficial effects or do not relate to the mechanism of action of insulin antagonists.

How well did you know this?
1
Not at all
2
3
4
5
Perfectly
20
Q

Which diagnostic test for Type 2 Diabetes provides information about the average blood glucose levels over the past three months?
A) Oral glucose tolerance test
B) Fasting blood glucose test
C) Postprandial blood glucose test
D) Glycated hemoglobin (HbA1c) test

A

Correct Answer: D) Glycated hemoglobin (HbA1c) test

Rationale: The HbA1c test measures the percentage of glycated hemoglobin in the blood, which reflects average blood glucose levels over approximately three months. The fasting blood glucose test measures glucose at a single point in time after an overnight fast, and the oral glucose tolerance test evaluates the body’s response to a glucose load over a couple of hours.

How well did you know this?
1
Not at all
2
3
4
5
Perfectly
21
Q

Which of the following is considered a primary site of insulin resistance in Type 2 Diabetes?
A) Pancreas
B) Heart
C) Skeletal muscle
D) Brain

A

Correct Answer: C) Skeletal muscle

Rationale: Insulin resistance primarily affects skeletal muscle, adipose tissue, and liver, as these tissues are major sites of glucose uptake and metabolism. The pancreas produces insulin, the heart is not a primary site of insulin action regarding glucose metabolism, and the brain is largely insulin-independent for glucose uptake.

How well did you know this?
1
Not at all
2
3
4
5
Perfectly
22
Q

Which glycated hemoglobin (HbA1c) level is indicative of diabetes according to the American Diabetes Association (ADA)?
A) 5.7%
B) 6.0%
C) 6.5%
D) 7.0%

A

Correct Answer: C) 6.5%

Rationale: An HbA1c level of 6.5% or higher on two separate tests indicates diabetes. Levels of 5.7-6.4% suggest prediabetes, and levels below 5.7% are considered normal.

How well did you know this?
1
Not at all
2
3
4
5
Perfectly
23
Q

What is the minimum fasting plasma glucose (FPG) level that, when confirmed on a subsequent day, suggests a diagnosis of diabetes?
A) 100 mg/dL (5.6 mmol/L)
B) 126 mg/dL (7.0 mmol/L)
C) 140 mg/dL (7.8 mmol/L)
D) 200 mg/dL (11.1 mmol/L)

A

Correct Answer: B) 126 mg/dL (7.0 mmol/L)

Rationale: The ADA specifies that a fasting plasma glucose level of 126 mg/dL (7.0 mmol/L) or higher, fasting defined as no caloric intake for at least 8 hours, suggests diabetes when confirmed on a subsequent day.

How well did you know this?
1
Not at all
2
3
4
5
Perfectly
24
Q

What is the required plasma glucose level threshold during an Oral Glucose Tolerance Test (OGTT) that confirms a diabetes diagnosis?
A) 140 mg/dL (7.8 mmol/L) after 2 hours
B) 180 mg/dL (10.0 mmol/L) after 2 hours
C) 200 mg/dL (11.1 mmol/L) after 2 hours
D) 200 mg/dL (11.1 mmol/L) after 1 hour

A

Correct Answer: C) 200 mg/dL (11.1 mmol/L) after 2 hours

Rationale: A 2-hour plasma glucose level of 200 mg/dL (11.1 mmol/L) or higher during an OGTT, with the test being carried out as per the World Health Organization’s protocol, confirms a diabetes diagnosis.

How well did you know this?
1
Not at all
2
3
4
5
Perfectly
25
Q

Under which condition might a random plasma glucose test be considered diagnostic for diabetes without requiring a second test for confirmation?
A) HbA1c of 6.0%
B) Presence of diabetes symptoms and a random plasma glucose ≥ 200 mg/dL
C) Fasting plasma glucose of 126 mg/dL
D) Two-hour plasma glucose of 140 mg/dL during an OGTT

A

Correct Answer: B) Presence of diabetes symptoms and a random plasma glucose ≥ 200 mg/dL

Rationale: The presence of classic symptoms of hyperglycemia or a hyperglycemic crisis, along with a random plasma glucose level of 200 mg/dL or higher, can be used to diagnose diabetes without the need for a second test.

How well did you know this?
1
Not at all
2
3
4
5
Perfectly
26
Q

Which laboratory method for HbA1c testing is preferred for diagnosing diabetes?
A) Laboratory method standardized to the International Federation of Clinical Chemistry (IFCC) assay
B) Laboratory method NGSP certified and standardized to the DCCT assay
C) Any laboratory method with an error margin within 10%
D) Laboratory methods that rely on point-of-care testing devices

A

Correct Answer: B) Laboratory method NGSP certified and standardized to the DCCT assay

Rationale: The ADA recommends that the laboratory method used for measuring HbA1c should be NGSP certified and standardized to the DCCT assay to ensure accuracy in diagnosing diabetes.

How well did you know this?
1
Not at all
2
3
4
5
Perfectly
27
Q

Metformin is considered the preferred initial pharmacotherapy for Type 2 Diabetes (DM2) primarily due to its effects on:
A) β-cell regeneration
B) Hepatic glucose production
C) Insulin secretion
D) Peripheral glucose uptake

A

Correct Answer:

D) Peripheral glucose uptake

Rationale:
Metformin primarily acts by enhancing glucose transport into tissues, improving peripheral insulin sensitivity, and decreasing hepatic glucose production. It does not stimulate insulin secretion nor directly cause β-cell regeneration.

How well did you know this?
1
Not at all
2
3
4
5
Perfectly
28
Q

Which of the following is a common side effect associated with sulfonylurea use in DM2 treatment?
A) Osteoporosis
B) Hypoglycemia
C) Hyperkalemia
D) Diarrhea

A

Correct Answer: B) Hypoglycemia

Rationale: Sulfonylureas increase insulin secretion, which can lead to hypoglycemia. Other side effects include weight gain and potential cardiac effects due to the risk of prolonged hypoglycemia.

How well did you know this?
1
Not at all
2
3
4
5
Perfectly
29
Q

Lifestyle interventions in DM2 management aim to improve insulin sensitivity by:
A) Reducing body muscle mass
B) Increasing adipose tissue deposition
C) Reducing hepatic and peripheral insulin resistance
D) Increasing the basal metabolic rate

A

Correct Answer:C) Reducing hepatic and peripheral insulin resistance

Rationale:
Lifestyle changes such as dietary adjustments and exercise leading to weight loss predominantly work by decreasing body fat, which improves hepatic and peripheral insulin sensitivity, reducing insulin resistance.

How well did you know this?
1
Not at all
2
3
4
5
Perfectly
30
Q

The long-term ineffectiveness of sulfonylureas in DM2 management is primarily due to:
A) Tolerance development to the drug
B) Exhaustion of pancreatic β-cell function
C) Receptor desensitization
D) Upregulation of hepatic glucose production

A

Correct Answer:
B) Exhaustion of pancreatic β-cell function

Rationale:
Sulfonylureas stimulate insulin secretion, which can over time lead to β-cell exhaustion. As the disease progresses and β-cell function declines, sulfonylureas become less effective.

How well did you know this?
1
Not at all
2
3
4
5
Perfectly
31
Q

Which additional benefit, aside from its antihyperglycemic effect, is associated with metformin therapy?
A) Increased high-density lipoprotein (HDL) cholesterol
B) Reduction in triglycerides (TGL) and low-density lipoprotein (LDL) cholesterol
C) Weight gain
D) Stimulation of appetite

A

Correct Answer:
B) Reduction in triglycerides (TGL) and low-density lipoprotein (LDL) cholesterol
Rationale:
Metformin has been shown to decrease levels of triglycerides and LDL cholesterol, which are beneficial cardiovascular effects beyond its glucose-lowering action. It is not typically associated with weight gain or appetite stimulation; in fact, it may aid in weight loss. Dr. David Sinclair’s magic longevity drug.

How well did you know this?
1
Not at all
2
3
4
5
Perfectly
32
Q

Which of the following antidiabetic agents has the potential to improve lipid profiles as an added benefit in DM2 treatment?
A. Metformin
B. Sulfonylureas
C. GLP-1 receptor agonists
D. Thiazolidinediones

A

Correct Answer: D. Thiazolidinediones

Rationale: Thiazolidinediones are known to have an additional benefit of improving lipid profiles, particularly pioglitazone, which can be advantageous for patients with DM2 and coexisting dyslipidemia.

How well did you know this?
1
Not at all
2
3
4
5
Perfectly
33
Q

In the context of DM2 treatment, which drug class carries a risk of fluid retention and heart failure?
A. DPP-4 inhibitors
B. GLP-1 receptor agonists
C. Thiazolidinediones
D. SGLT-2 inhibitors

A

Correct Answer: C. Thiazolidinediones

Rationale: Thiazolidinediones can lead to fluid retention and are associated with an increased risk of heart failure, which is a significant consideration in the management of DM2, especially in patients with a history of heart failure or other cardiovascular risks.

How well did you know this?
1
Not at all
2
3
4
5
Perfectly
34
Q

For a patient with DM2 and a high risk of cardiovascular disease (CVD), which medication class may offer cardiovascular benefits beyond glycemic control?
A. Sulfonylureas
B. GLP-1 receptor agonists
C. DPP-4 inhibitors
D. α-glucosidase inhibitors

A

Correct Answer: B. GLP-1 receptor agonists

Rationale: GLP-1 receptor agonists, such as semaglutide and dulaglutide, have been shown to reduce major adverse cardiovascular events, making them suitable for DM2 patients with high cardiovascular risk.

How well did you know this?
1
Not at all
2
3
4
5
Perfectly
35
Q

Which antidiabetic medication is associated with beneficial outcomes in DM2 patients with nephropathy?
A. Sulfonylureas
B. SGLT-2 inhibitors
C. α-glucosidase inhibitors
D. Insulin

A

Correct Answer: B. SGLT-2 inhibitors
Rationale: SGLT-2 inhibitors have demonstrated renal protective effects, with evidence showing improved renal outcomes in patients with DM2 and nephropathy.

And acute kidney injury as a disadvantage.

How well did you know this?
1
Not at all
2
3
4
5
Perfectly
36
Q

Which insulin type is primarily used to manage postprandial blood glucose spikes in DM1 and DM2?
A. Rapid acting
B. Short acting
C. Basal
D. Long acting

A

Correct Answer: A. Rapid acting

Rationale: Rapid-acting insulins, such as Lispro and Aspart, are designed to control glucose surges after meals, providing fast glucose control at mealtimes.

How well did you know this?
1
Not at all
2
3
4
5
Perfectly
37
Q

Chronic exposure to hypoglycemic episodes in DM treatment can lead to a state where the patient:
A. Experiences enhanced perception of hypoglycemia.
B. No longer has autonomic or neuroglycopenic symptoms of hypoglycemia.
C. Is at decreased risk for neuroglycopenia.
D. Develops irreversible neurologic damage.

A

Correct Answer: B. No longer has autonomic or neuroglycopenic symptoms of hypoglycemia.

Rationale: Repeated hypoglycemic episodes can lead to hypoglycemia unawareness, a condition where the patient becomes desensitized to low blood sugar levels and does not experience the typical warning symptoms, increasing the risk for severe hypoglycemia.

How well did you know this?
1
Not at all
2
3
4
5
Perfectly
38
Q

In the event of hypoglycemia with altered consciousness, which route of administration for glucose is appropriate?
A. Oral (PO) only
B. Subcutaneous (SQ) only
C. Intramuscular (IM) or intravenous (IV)
D. Transdermal

A

Correct Answer: C. Intramuscular (IM) or intravenous (IV)

Rationale: When a patient has altered consciousness due to hypoglycemia, they cannot safely take oral glucose. Therefore, glucose should be administered intramuscularly or intravenously to rapidly increase blood glucose levels. Subcutaneous injection is not used in acute settings due to slower absorption.

How well did you know this?
1
Not at all
2
3
4
5
Perfectly
39
Q

What factors can exacerbate the risk of hypoglycemia in patients treated with insulin?
A. Alcohol consumption and use of ACE inhibitors
B. Vigorous exercise and high carbohydrate intake
C. Use of anabolic steroids and thiazide diuretics
D. Supplementation with omega-3 fatty acids and vitamin D

A

Correct Answer: A. Alcohol consumption and use of ACE inhibitors

Rationale: Alcohol (ETOH) can inhibit gluconeogenesis, and ACE inhibitors can increase insulin sensitivity, both potentially exacerbating the risk of hypoglycemia. The other options are less likely to cause hypoglycemia or may, in fact, be protective against it.

How well did you know this?
1
Not at all
2
3
4
5
Perfectly
40
Q

Which insulin has the longest duration of action, potentially providing basal coverage for a full day or longer?
A. Lente
B. Detemir
C. Glargine (U-300)
D. Aspart (NovoLog)

A

Correct Answer: C. Glargine (U-300)

Rationale: Ultra-long-acting insulins, such as Glargine U-300, have an extended duration of action that can last up to 24 hours or more, providing a steady level of insulin, which is ideal for basal coverage. (note basal technically means intermediate acting)

How well did you know this?
1
Not at all
2
3
4
5
Perfectly
41
Q

Considering the pharmacodynamics of rapid-acting insulins, what is the most appropriate timing for administration in relation to meals?
A. 30-60 minutes before meals
B. At the start of meals
C. 15-20 minutes after starting a meal
D. 1-2 hours post-meal

A

Correct Answer: B. At the start of meals

Rationale: Rapid-acting insulins like aspart and lispro are designed to be taken at mealtime because of their quick onset, matching the body’s immediate insulin needs post ingestion of food. 10-15 min onset, peak 1 hour.

How well did you know this?
1
Not at all
2
3
4
5
Perfectly
42
Q

When transitioning from once-daily long-acting insulin to twice-daily intermediate-acting insulin, which characteristic of intermediate-acting insulins must be considered?
A. Their peak action time
B. Their rapid onset of action
C. Their lack of a peak
D. Their short duration of action

A

Correct Answer: A. Their peak action time

Rationale: Intermediate-acting insulins, such as NPH, have a peak action time where their effect is maximal. When switching from a long-acting insulin that provides a constant level of insulin, this peak must be considered to avoid hypoglycemia.

How well did you know this?
1
Not at all
2
3
4
5
Perfectly
43
Q

In the pharmacokinetic profile of insulin, what does the term ‘peak’ refer to?
A. The time it takes for insulin to start working
B. The time it takes for insulin to be completely absorbed
C. The time at which insulin is at the highest concentration in the bloodstream
D. The time at which insulin is no longer effective

A

Correct Answer: C. The time at which insulin is at the highest concentration in the bloodstream

Rationale: The peak of insulin action refers to the period when the insulin is at its maximum strength in terms of lowering blood glucose. This is an important consideration for timing insulin administration and anticipating potential hypoglycemia.

How well did you know this?
1
Not at all
2
3
4
5
Perfectly
44
Q

For a patient requiring a quick reduction of high blood glucose levels, which insulin type would be the most appropriate choice?
A. Intermediate-acting
B. Short-acting
C. Long-acting
D. Rapid-acting

A

Correct Answer: D. Rapid-acting

Rationale: Rapid-acting insulins, like lispro or aspart, have a quick onset of action, usually within 10-15 minutes, making them ideal for correcting high blood glucose levels shortly after administration.

regular insulin is short acting w/ an onset time of 30 min, peak of 2-4 hr

How well did you know this?
1
Not at all
2
3
4
5
Perfectly
45
Q

What is the primary mechanism leading to hypovolemia in diabetic ketoacidosis (DKA)?
A. Insulin-induced hyperhydration
B. Glucagon-mediated renal retention
C. Osmotic diuresis due to hyperglycemia
D. Reduced thirst response

A

Correct Answer: C. Osmotic diuresis due to hyperglycemia

Rationale: In DKA, the high glucose levels exceed the renal threshold for reabsorption, leading to glucosuria which causes osmotic diuresis, and consequently, hypovolemia.

How well did you know this?
1
Not at all
2
3
4
5
Perfectly
46
Q

Which condition is a common precipitating factor for the development of DKA in individuals with DM1?
A. Physical trauma
B. Electrolyte imbalance
C. Infection or illness
D. Overuse of insulin

A

Correct Answer: C. Infection or illness

Rationale: Infections or illness can increase stress hormone levels, which antagonize insulin action, and may precipitate the onset of DKA, especially in patients with DM1.

How well did you know this?
1
Not at all
2
3
4
5
Perfectly
47
Q

The metabolic state in DKA is characterized by increased production of which substances?
A. Glucose and insulin
B. Ketone bodies and bicarbonate
C. Free fatty acids and ketone bodies
D. Glycogen and carbon dioxide

A

Correct Answer: C. Free fatty acids and ketone bodies

Rationale: DKA involves the excessive production of ketone bodies due to increased lipolysis and free fatty acid metabolism in the liver, driven by insulin deficiency and counterregulatory hormone excess.

How well did you know this?
1
Not at all
2
3
4
5
Perfectly
48
Q

In the diagnosis of DKA, which laboratory finding is indicative of an acidotic state?
A. Serum pH greater than 7.45
B. Bicarbonate (HCO₃⁻) levels above 18 mEq/L
C. Serum pH less than 7.3
D. Elevated serum osmolarity

A

Correct Answer: C. Serum pH less than 7.3

Rationale: A serum pH less than 7.3 reflects an acidotic state, which is a hallmark of DKA due to the accumulation of ketoacids in the bloodstream.

How well did you know this?
1
Not at all
2
3
4
5
Perfectly
49
Q

What is the initial pharmacological intervention in the management of diabetic ketoacidosis (DKA)?
A. Oral hypoglycemic agents
B. Subcutaneous long-acting insulin
C. Intravenous short-acting insulin bolus
D. Intravenous bicarbonate infusion

A

Correct Answer: C. Intravenous short-acting insulin bolus

Rationale: The initial treatment for DKA involves administering an intravenous bolus of short-acting insulin, usually regular insulin, at a dose of 0.1 units/kg, followed by a continuous infusion to lower blood glucose levels in a controlled manner.

How well did you know this?
1
Not at all
2
3
4
5
Perfectly
50
Q

Which electrolyte is commonly supplemented during DKA treatment due to losses from osmotic diuresis?
A. Calcium
B. Chloride
C. Potassium
D. Carbonate

A

Correct Answer: C. Potassium

Rationale: Potassium is often supplemented during the treatment of DKA because insulin therapy promotes the uptake of potassium into cells and the osmotic diuresis associated with hyperglycemia leads to significant potassium losses.

How well did you know this?
1
Not at all
2
3
4
5
Perfectly
51
Q

What is the potential risk of rapid glucose correction without addressing changes in serum sodium during DKA treatment?
A. Hyperosmolar hyperglycemic state
B. Hypokalemic hypochloremic alkalosis
C. Hypernatremia
D. Cerebral edema

A

Correct Answer: D. Cerebral edema

Rationale: Rapid correction of hyperglycemia without the simultaneous correction of sodium can lead to cerebral edema. During treatment, it is crucial to monitor serum glucose and sodium levels and adjust the rate of glucose correction to reduce this risk.

How well did you know this?
1
Not at all
2
3
4
5
Perfectly
52
Q

The administration of sodium bicarbonate in the treatment of DKA is primarily aimed at:
A. Enhancing urinary excretion of ketones
B. Correcting metabolic acidosis
C. Increasing blood glucose reabsorption
D. Preventing hyponatremia

A

Correct Answer: B. Correcting metabolic acidosis

Rationale: Sodium bicarbonate is used in DKA treatment to correct the metabolic acidosis that results from the overproduction and accumulation of ketoacids in the blood.

How well did you know this?
1
Not at all
2
3
4
5
Perfectly
53
Q

Hyperglycemic Hyperosmolar Syndrome (HHS) is primarily observed in patients with:
A. Type 1 Diabetes Mellitus under the age of 30
B. Type 2 Diabetes Mellitus over the age of 60
C. Gestational diabetes
D. Cystic fibrosis-related diabetes

A

Correct Answer: B. Type 2 Diabetes Mellitus over the age of 60

Rationale: HHS typically occurs in older individuals with Type 2 Diabetes Mellitus, evolving gradually over days to weeks, and is characterized by severe hyperglycemia, hyperosmolarity, and dehydration.

How well did you know this?
1
Not at all
2
3
4
5
Perfectly
54
Q

Which clinical manifestation is not typically associated with Hyperglycemic Hyperosmolar Syndrome (HHS)?
A. Ketosis and fruity breath odor
B. Tachycardia and hypotension
C. Polyuria and polydipsia
D. Altered mental status and severe dehydration

A

Correct Answer: A. Ketosis and fruity breath odor

Rationale: Unlike diabetic ketoacidosis (DKA), HHS usually does not involve significant ketosis or the distinctive fruity breath odor caused by ketones, although patients can have some degree of acidosis.

How well did you know this?
1
Not at all
2
3
4
5
Perfectly
55
Q

The primary treatment for Hyperglycemic Hyperosmolar Syndrome includes:
A. Aggressive insulin therapy without fluid replacement
B. Oral rehydration therapy and dietary modifications
C. Fluid resuscitation and gradual insulin therapy
D. Immediate correction of hyperglycemia with rapid insulin bolus

A

C. Fluid resuscitation and gradual insulin therapy

Rationale: The initial management of HHS focuses on fluid replacement to address severe dehydration and hyperosmolarity, followed by gradual insulin administration to correct hyperglycemia, avoiding rapid shifts that may lead to complications such as cerebral edema.

fluid resuscitation, insulin bolus + infusion, e-lytes

How well did you know this?
1
Not at all
2
3
4
5
Perfectly
56
Q

What is the significant risk associated with hyperosmolarity in Hyperglycemic Hyperosmolar Syndrome?
A. Hypokalemia
B. Peripheral neuropathy
C. Cerebral edema
D. Coma

A

D. Coma

How well did you know this?
1
Not at all
2
3
4
5
Perfectly
57
Q

What percentage of individuals with Type 1 Diabetes Mellitus (DM1) are at risk of developing end-stage renal disease (ESRD)?
A. 5-10%
B. 15-25%
C. 30-40%
D. 45-50%

A

Correct Answer: C. 30-40%

Rationale: Diabetic nephropathy, a serious complication of diabetes, can progress to ESRD, with approximately 30-40% of individuals with DM1 being at risk. 5-10% develop ESRD

How well did you know this?
1
Not at all
2
3
4
5
Perfectly
58
Q

Which class of medication is indicated to slow the progression of proteinuria and the decline of the glomerular filtration rate (GFR) in diabetic nephropathy?
A. Calcium channel blockers
B. Angiotensin-converting enzyme inhibitors (ACE-I’s)
C. Beta-blockers
D. Diuretics

A

Correct Answer: B. Angiotensin-converting enzyme inhibitors (ACE-I’s)

Rationale: ACE inhibitors are used to slow the progression of proteinuria and the rate of GFR decline in patients with diabetic nephropathy. They have renoprotective effects that go beyond their ability to lower blood pressure.

How well did you know this?
1
Not at all
2
3
4
5
Perfectly
59
Q

What is a potential treatment for end-stage renal disease (ESRD) secondary to diabetic nephropathy?
A. Lifelong dialysis only
B. Hemodialysis (HD) and peritoneal dialysis (PD)
C. Insulin therapy alone
D. Strict glycemic control with diet

A

Correct Answer: B. Hemodialysis (HD) and peritoneal dialysis (PD)

Rationale: ESRD may be treated with various forms of dialysis, including hemodialysis and peritoneal dialysis, or ultimately with a kidney transplant - can combine w/ pancreas to prevent recurrence. Insulin therapy and strict glycemic control are important but not sufficient as sole treatments for ESRD.

How well did you know this?
1
Not at all
2
3
4
5
Perfectly
59
Q

At what threshold of GFR are patients with diabetic nephropathy typically unable to clear potassium effectively, leading to hyperkalemia?
A. GFR > 60 mL/min
B. GFR 30-59 mL/min
C. GFR 15-29 mL/min
D. GFR < 15-20 mL/min

A

Correct Answer: D. GFR < 15-20 mL/min

Rationale: When the GFR falls below 15-20 mL/min, the kidneys’ ability to excrete potassium is significantly impaired, resulting in hyperkalemia and metabolic acidosis.

How well did you know this?
1
Not at all
2
3
4
5
Perfectly
60
Q

Diabetic peripheral neuropathy commonly presents with which of the following symptoms initially?
A. Numbness in the upper extremities
B. Pain and temperature perception changes in the toes/feet
C. Proprioception loss in the hands
D. Motor weakness in the proximal muscle groups

A

Correct Answer: B. Pain and temperature perception changes in the toes/feet

Rationale: Diabetic peripheral neuropathy typically begins with sensory deficits in the distal extremities, notably the toes and feet, often presenting as changes in pain and temperature perception.

How well did you know this?
1
Not at all
2
3
4
5
Perfectly
61
Q

Which therapeutic intervention is considered a cornerstone in the management of diabetic peripheral neuropathy?
A. High-dose vitamin B12 supplementation
B. Optimal glycemic control
C. Corticosteroid injections
D. Continuous opioid analgesia

A

Correct Answer: B. Optimal glycemic control

Rationale: Optimal glycemic control is essential in the management of diabetic peripheral neuropathy, as it can slow the progression of neuropathy and potentially improve symptoms. Other treatments, such as NSAIDs, antidepressants, and anticonvulsants, can be used to manage pain and discomfort.

How well did you know this?
1
Not at all
2
3
4
5
Perfectly
62
Q

In diabetic retinopathy, which of the following changes is a direct consequence of microvascular damage?
A. Retinal detachment
B. Increased permeability and microaneurysm formation
C. Cataract formation
D. Ocular muscle paralysis

A

Correct Answer: B. Increased permeability and microaneurysm formation

Rationale: Diabetic retinopathy is characterized by microvascular changes that lead to increased permeability, vessel occlusion, and microaneurysms in the retina, directly impacting vision.

How well did you know this?
1
Not at all
2
3
4
5
Perfectly
63
Q

The development of ulcers in diabetic peripheral neuropathy is primarily due to:
A. Poorly fitting footwear
B. Unnoticed mechanical and traumatic injuries
C. Frequent hypoglycemic episodes
D. Excessive exercise

A

Correct Answer: B. Unnoticed mechanical and traumatic injuries

Rationale: Ulcers in diabetic peripheral neuropathy often result from mechanical and traumatic injuries that go unnoticed due to sensory deficits. These ulcers can lead to significant morbidity due to recurrent infections and risk of amputation.

Loss of small nerve fibers decrease pain/temp perception, causing neuropathic pain

How well did you know this?
1
Not at all
2
3
4
5
Perfectly
64
Q

Autonomic neuropathy in diabetes may lead to cardiovascular abnormalities such as:
A. Increased heart rate variability
B. Hypertensive urgency
C. Orthostatic hypotension
D. Bradycardia at rest

A

Correct Answer: C. Orthostatic hypotension

Rationale: Autonomic neuropathy can damage the vasomotor fibers and baroreceptors, leading to ineffective cardiovascular responses such as orthostatic hypotension, where there’s a significant drop in blood pressure upon standing.

How well did you know this?
1
Not at all
2
3
4
5
Perfectly
65
Q

One of the main gastrointestinal symptoms of diabetic autonomic neuropathy is:
A. Diarrhea due to increased motility
B. Constipation from reduced intestinal secretions
C. Gastroparesis resulting from decreased gastric motility
D. Increased gastric acid secretion leading to ulcers

A

Correct Answer: C. Gastroparesis resulting from decreased gastric motility

Rationale: Diabetic autonomic neuropathy can cause a reduction in gastric secretions and motility, often leading to gastroparesis, which is characterized by delayed gastric emptying and associated symptoms like nausea, vomiting, and early satiety.

How well did you know this?
1
Not at all
2
3
4
5
Perfectly
66
Q

The therapeutic approach to managing gastrointestinal complications of diabetic autonomic neuropathy includes:
A. Large, less frequent meals
B. Use of prokinetics and large meals
C. Small, frequent meals and prokinetics
D. Complete fasting

A

Correct Answer: C. Small, frequent meals and prokinetics

Rationale: Treatment of gastroparesis and related symptoms involves improved glucose control, eating small, frequent meals to facilitate gastric emptying, and the use of prokinetic drugs to enhance gastrointestinal motility.

How well did you know this?
1
Not at all
2
3
4
5
Perfectly
67
Q

In diabetic patients, why is a preoperative cardiac stress test recommended in the presence of multiple cardiac risk factors and poor exercise tolerance?
A. To evaluate for pulmonary hypertension
B. To assess for the presence of silent myocardial ischemia
C. To determine baseline heart rate variability
D. To evaluate for exercise-induced bronchospasm

A

Correct Answer: B. To assess for the presence of silent myocardial ischemia

Rationale: Diabetic autonomic neuropathy may blunt the typical symptoms of myocardial ischemia. Therefore, a stress test is indicated to detect any silent ischemic heart disease in diabetic patients, particularly those with additional cardiac risk factors or poor exercise tolerance.

How well did you know this?
1
Not at all
2
3
4
5
Perfectly
68
Q

Why is meticulous attention to hydration status critical in diabetic patients undergoing surgery?
A. To prevent hypotension and reduce the risk of cerebral edema
B. To decrease the risk of postoperative infections
C. To avoid nephrotoxins and preserve renal blood flow (RBF)
D. To ensure adequate gastrointestinal motility

A

Correct Answer: C. To avoid nephrotoxins and preserve renal blood flow (RBF)

Rationale: Diabetic patients are at an increased risk for nephropathy. Adequate hydration helps to preserve renal function by maintaining renal blood flow and avoiding additional renal insults from nephrotoxic agents.

How well did you know this?
1
Not at all
2
3
4
5
Perfectly
69
Q

What is a primary reason for the increased risk of perioperative dysrhythmia in diabetic patients with autonomic neuropathy?
A. They are more sensitive to anesthetic agents.
B. There is a reduction in normal heart rate variability.
C. They require higher doses of beta-blockers.
D. They have an inherent predisposition to atrial fibrillation.

A

Correct Answer: B. There is a reduction in normal heart rate variability.

Rationale: Autonomic neuropathy affects cardiovascular autonomic fibers, leading to decreased heart rate variability, which predisposes patients to perioperative cardiac dysrhythmias and hypotension.

How well did you know this?
1
Not at all
2
3
4
5
Perfectly
70
Q

How does gastroparesis in diabetic patients affect anesthetic management?
A. It necessitates the use of prokinetic agents intraoperatively.
B. It decreases the risk of aspiration.
C. It increases the risk of aspiration pneumonia, regardless of nil per os (NPO) status.
D. It requires the administration of higher doses of antiemetics.

A

Correct Answer: C. It increases the risk of aspiration pneumonia, regardless of nil per os (NPO) status.

Rationale: Gastroparesis can delay gastric emptying, leading to increased gastric contents and volume, which increases the risk of aspiration during anesthesia, regardless of fasting status.

How well did you know this?
1
Not at all
2
3
4
5
Perfectly
71
Q

What is the characteristic triad of symptoms for diagnosing insulinoma, commonly known as Whipple’s triad?
A. Hyperglycemia with fasting, glucose >200 w/sx, sx relief with insulin
B. Hypoglycemia with fasting, glucose <50 w/sx, sx relief with glucose
C. Epigastric pain, jaundice, and weight loss
D. Diarrhea, dermatitis, and dementia

A

Correct Answer: B. Hypoglycemia with fasting, glucose <50 w/sx, sx relief with glucose

Rationale: The Whipple’s triad includes symptoms of hypoglycemia with fasting, documentation of low plasma glucose (<50 mg/dL) with symptoms, and relief of symptoms after administration of glucose, which is indicative of an insulinoma.

How well did you know this?
1
Not at all
2
3
4
5
Perfectly
72
Q

In the management of an insulinoma, what is the rationale for using diazoxide preoperatively?
A. To increase insulin secretion and prevent hypoglycemia
B. To inhibit insulin release from beta cells and prevent hypoglycemia
C. To stimulate appetite and prevent weight loss
D. To reduce tumor size before surgery

A

Correct Answer: B. To inhibit insulin release from beta cells and prevent hypoglycemia

Rationale: Diazoxide is used preoperatively in patients with insulinoma because it inhibits insulin secretion from pancreatic beta cells, helping to prevent hypoglycemia before the tumor can be surgically removed.

How well did you know this?
1
Not at all
2
3
4
5
Perfectly
73
Q

What is the most definitive treatment for insulinoma?
A. Chemotherapy
B. Radiation therapy
C. Long-term medication with diazoxide
D. Surgery

A

Correct Answer: D. Surgery

Rationale: Surgical removal of the insulinoma is curative, as these tumors are typically benign and isolated. Surgery addresses the underlying cause of the hypoglycemia by removing the tumor.

Hypoglycemia can occur intra-op, followed by hyperglycemia once tumor removed. Tight glycemic control is paramount.

How well did you know this?
1
Not at all
2
3
4
5
Perfectly
74
Q

What is the primary functional unit within the thyroid gland responsible for thyroid hormone production?
A. Thyroid follicles
B. Parafollicular C cells
C. Parathyroid glands
D. Adrenergic synapses

A

Correct Answer: A. Thyroid follicles

Rationale: The thyroid follicles, composed of a layer of epithelial cells surrounding a lumen filled with colloid (primarily thyroglobulin), are the fundamental functional units of the thyroid gland where thyroid hormones are synthesized and stored.

How well did you know this?
1
Not at all
2
3
4
5
Perfectly
75
Q

Which hormone is secreted by the parafollicular C cells of the thyroid gland?
A. Thyroxine (T4)
B. Triiodothyronine (T3)
C. Parathyroid hormone (PTH)
D. Calcitonin

A

Correct Answer: D. Calcitonin

Rationale: Parafollicular C cells of the thyroid gland produce calcitonin, a hormone involved in the regulation of calcium and phosphate levels in the blood, counteracting the actions of parathyroid hormone (PTH).

How well did you know this?
1
Not at all
2
3
4
5
Perfectly
76
Q

In the context of thyroid surgery, why is the identification and preservation of the recurrent laryngeal nerve of utmost importance?
A. It provides sensory innervation to the thyroid gland.
B. It is responsible for vocal cord movement and protection of the airway.
C. It regulates parathyroid hormone secretion.
D. It controls the sympathetic response of the thyroid gland.

A

Correct Answer: B. It is responsible for vocal cord movement and protection of the airway.

Rationale: The recurrent laryngeal nerve innervates the intrinsic muscles of the larynx, controlling their movement. Injury to this nerve during thyroid surgery can lead to vocal cord paralysis and compromised airway protection.

external motor branch of superior laryngeal nerve are in close proximity to the thyroid

How well did you know this?
1
Not at all
2
3
4
5
Perfectly
77
Q

What enzyme is crucial for the synthesis of thyroid hormones by catalyzing the binding of iodide to thyroglobulin?
A. Thyroid peroxidase
B. Thyroxine-binding globulin
C. Iodinase
D. Tyrosinase

A

Correct Answer: A. Thyroid peroxidase

Rationale: Thyroid peroxidase (TPO) is the key enzyme in the thyroid hormone synthesis process, which catalyzes the iodination of tyrosine residues on thyroglobulin and the coupling of iodotyrosines to form thyroid hormones T3 and T4.

How well did you know this?
1
Not at all
2
3
4
5
Perfectly
78
Q

Which of the following proteins binds the majority of circulating T4 in the bloodstream?
A. Albumin
B. Prealbumin
C. Thyroxine-binding globulin
D. Globulin

A

Correct Answer: C. Thyroxine-binding globulin

Rationale: Thyroxine-binding globulin (TBG) is the primary carrier protein for thyroid hormones in the blood, binding up to 80% of T4. Prealbumin (transthyretin) and albumin also bind thyroid hormones, but to a lesser extent.

How well did you know this?
1
Not at all
2
3
4
5
Perfectly
79
Q

What is the primary role of thyroid hormones in the body?
A. To regulate calcium homeostasis
B. To stimulate red blood cell production
C. To regulate metabolism and stimulate tissue development
D. To modulate fluid balance and electrolyte transport

A

Correct Answer: C. To regulate metabolism and stimulate tissue development

Rationale: Thyroid hormones play a pivotal role in metabolism, influencing virtually all metabolic processes. They are crucial for growth and development, tissue function, and the metabolism of proteins, carbohydrates, and fats.

How well did you know this?
1
Not at all
2
3
4
5
Perfectly
80
Q

Which process is necessary for the conversion of dietary iodine into a form that can be utilized for thyroid hormone synthesis?
A. Oxidation to iodide in the GI tract
B. Methylation in the liver
C. Sulfation in the thyroid gland
D. Phosphorylation in the bloodstream

A

Correct Answer: A. Oxidation to iodide in the GI tract

Rationale: Dietary iodine is converted into iodide through an oxidation process in the gastrointestinal tract. This iodide is then rapidly absorbed into the bloodstream and transported to the thyroid follicular cells to be used in thyroid hormone synthesis.

How well did you know this?
1
Not at all
2
3
4
5
Perfectly
81
Q

What is the significance of the thyroid gland’s large store of hormones and low turnover rate?
A. It enables rapid responses to stress.
B. It provides a buffer against dietary excess of iodine.
C. It allows for immediate secretion of hormones with sympathetic stimulation.
D. It protects against hormone depletion if synthesis is impaired.

A

Correct Answer: D. It protects against hormone depletion if synthesis is impaired.

Rationale: The thyroid gland’s large store of hormones and low turnover rate provide a reservoir that can maintain normal levels of thyroid hormones in the bloodstream for a period of time even if new hormone synthesis is disrupted, thus protecting against hypothyroidism.

How well did you know this?
1
Not at all
2
3
4
5
Perfectly
82
Q

What hormone is responsible for stimulating the release of thyroid-stimulating hormone (TSH) from the anterior pituitary?
A. Thyroid-releasing hormone (TRH)
B. Corticotropin-releasing hormone (CRH)
C. Growth hormone-releasing hormone (GHRH)
D. Gonadotropin-releasing hormone (GnRH)

A

Correct Answer: A. Thyroid-releasing hormone (TRH)

Rationale: Thyrotropin-releasing hormone (TRH) is secreted by the hypothalamus and stimulates the anterior pituitary to release thyroid-stimulating hormone (TSH), which in turn stimulates the thyroid gland to produce thyroid hormones T3 and T4.

How well did you know this?
1
Not at all
2
3
4
5
Perfectly
83
Q

How does the hypothalamic-pituitary-thyroid axis regulate thyroid hormone levels through a feedback system?
A. TSH levels increase in response to high T3 and T4 levels.
B. High T3 and T4 levels stimulate further release of TRH.
C. Low levels of circulating T3 and T4 inhibit TSH release.
D. High levels of T3 and T4 inhibit TSH release.

A

Correct Answer: D. High levels of T3 and T4 inhibit TSH release.

Rationale: The production of thyroid hormones is regulated by a negative feedback loop. When the levels of T3 and T4 are high, they inhibit the release of TRH from the hypothalamus and TSH from the anterior pituitary, thus reducing thyroid hormone synthesis.

How well did you know this?
1
Not at all
2
3
4
5
Perfectly
84
Q

What physiological changes occur in the thyroid gland when TSH levels are decreased?
A. Increased T3 and T4 synthesis and increased vascularization
B. Decreased T3 and T4 synthesis and increased follicular cell size
C. Increased T3 and T4 synthesis and decreased vascularization
D. Decreased T3 and T4 synthesis and decreased follicular cell size

A

Correct Answer: D. Decreased T3 and T4 synthesis and decreased follicular cell size

Rationale: A decrease in TSH results in decreased synthesis of T3 and T4, as well as reduced follicular cell size and gland vascularization, due to the reduced stimulation of the thyroid gland.

How well did you know this?
1
Not at all
2
3
4
5
Perfectly
85
Q

What is the role of thyroid-stimulating hormone (TSH) in the thyroid gland?
A. To reduce the secretion of T3 and T4
B. To bind to receptors on thyroid cell membranes and enhance T3 and T4 secretion
C. To initiate the autoregulatory mechanism of the thyroid gland
D. To directly inhibit the synthesis of thyroglobulin

A

Correct Answer: B. To bind to receptors on thyroid cell membranes and enhance T3 and T4 secretion

Rationale: TSH binds to specific receptors on the membranes of thyroid cells and stimulates the synthesis and secretion of the thyroid hormones T3 and T4.

How well did you know this?
1
Not at all
2
3
4
5
Perfectly
86
Q

What mechanism ensures consistent thyroid hormone levels within the body?
A. Positive feedback control of TSH
B. Thyroid autoregulation
C. Hypothalamic suppression
D. Pituitary inhibition

A

Correct Answer: B. Thyroid autoregulation

Rationale: The thyroid gland has an autoregulatory mechanism that adjusts the synthesis and release of thyroid hormones to maintain consistent hormone levels, independent of TSH regulation.

How well did you know this?
1
Not at all
2
3
4
5
Perfectly
87
Q

What is the primary purpose of the TRH stimulation test?
A. To determine the responsiveness of the thyroid gland to TSH
B. To assess the functional state of the pituitary gland in secreting TSH
C. To measure the level of thyroid hormones in the blood
D. To evaluate the presence of thyroid autoantibodies

A

Correct Answer: B. To assess the functional state of the pituitary gland in secreting TSH

Rationale: The TRH stimulation test is used to evaluate the function of the pituitary gland by administering thyrotropin-releasing hormone and measuring the subsequent secretion of TSH, thereby assessing the pituitary’s capacity to respond to hypothalamic input.

How well did you know this?
1
Not at all
2
3
4
5
Perfectly
88
Q

What is the significance of the normal range of TSH in thyroid function tests?
A. It indicates the maximum allowable TSH concentration in the blood.
B. It suggests the threshold for initiating thyroid hormone replacement therapy.
C. It reflects the sensitivity of the hypothalamic-pituitary-thyroid axis to small changes in thyroid hormone levels.
D. It defines the level at which thyroid hormones inhibit TSH secretion.

A

Correct Answer: C. It reflects the sensitivity of the hypothalamic-pituitary-thyroid axis to small changes in thyroid hormone levels.

Rationale: The normal range of TSH (0.4-5.0 milliunits/L) reflects the tight regulation of thyroid function by the hypothalamic-pituitary-thyroid axis, where even small changes in thyroid hormone levels can lead to significant changes in TSH secretion

How well did you know this?
1
Not at all
2
3
4
5
Perfectly
89
Q

In the context of thyroid testing, what is indicated by a “hot” nodule on a thermal thyroid scan?
A. The nodule is non-functioning.
B. The nodule is cancerous.
C. The nodule is producing an excess of thyroid hormone.
D. The nodule has reduced blood flow.

A

Correct Answer: C. The nodule is producing an excess of thyroid hormone.

Rationale: In thermal imaging of the thyroid, a “hot” nodule is one that shows increased uptake of the radioactive tracer, indicating “hyperfunctioning” tissue that is producing an excess of thyroid hormone.

warm is normal functioning

How well did you know this?
1
Not at all
2
3
4
5
Perfectly
90
Q

What is the utility of ultrasonography (US) in the evaluation of thyroid lesions?
A. To identify the specific type of thyroid cancer
B. To determine the biochemical activity of a thyroid nodule
C. To determine whether a thyroid lesion is cystic, solid, or mixed in composition
D. To measure the concentration of thyroid hormones within a nodule

A

Correct Answer: C. To determine whether a thyroid lesion is cystic, solid, or mixed in composition

Rationale: Ultrasonography is highly accurate (90-95%) in differentiating the physical composition of thyroid lesions, which is critical for determining the nature of the lesion and guiding further diagnostic or therapeutic steps.

How well did you know this?
1
Not at all
2
3
4
5
Perfectly
91
Q

Which of the following conditions is a common cause of hyperthyroidism?
A. Hashimoto’s thyroiditis
B. Graves’ disease
C. Thyroid lymphoma
D. De Quervain’s thyroiditis

A

Correct Answer: B. Graves’ disease

Rationale: Graves’ disease is an autoimmune disorder and one of the most common causes of hyperthyroidism, characterized by the overproduction of thyroid hormones.

major diseases: Graves disease
toxic multinodular goiter
toxic adenoma

How well did you know this?
1
Not at all
2
3
4
5
Perfectly
92
Q

What are the clinical manifestations of hyperthyroidism associated with its effect on metabolism?
A. Bradycardia and cold intolerance
B. Hypotension and hypoactivity
C. Sweating, heat intolerance, and fatigue
D. Weight gain and reduced appetite

A

Correct Answer: C. Sweating, heat intolerance, and fatigue

Rationale: The symptoms of hyperthyroidism, such as sweating, heat intolerance, and fatigue, are due to the hypermetabolic state induced by excessive thyroid hormones.

How well did you know this?
1
Not at all
2
3
4
5
Perfectly
93
Q

How does triiodothyronine (T3) affect the cardiovascular system in hyperthyroidism?
A. Decreases heart rate and myocardial contractility
B. Increases systemic vascular resistance
C. Acts directly on the myocardium and peripheral vasculature to cause cardiovascular responses
D. Slows down the conduction of electrical impulses in the heart

A

Correct Answer: C. Acts directly on the myocardium and peripheral vasculature to cause cardiovascular responses

Rationale: T3 has a direct effect on the myocardium and peripheral vasculature, leading to increased heart rate, contractility, and vasodilation, contributing to the cardiovascular manifestations of hyperthyroidism such as tachycardia and palpitations.

How well did you know this?
1
Not at all
2
3
4
5
Perfectly
94
Q

Which neurologic symptom is typical in a hyperthyroid patient?
A. Hypoactive deep tendon reflexes
B. Fine tremor of hands
C. Paresthesia
D. Generalized seizures

A

Correct Answer: B. Fine tremor of hands

Rationale: Fine tremor of the hands is a common neurologic manifestation of hyperthyroidism, often associated with the hypermetabolic state that accelerates various physiologic processes.

How well did you know this?
1
Not at all
2
3
4
5
Perfectly
95
Q

In the context of hyperthyroidism, what does a “hyperdynamic” cardiac state imply?
A. Decreased heart rate and myocardial contractility
B. Reduction in cardiac output
C. Increased heart rate and enhanced contractility
D. Myocardial hypertrophy and bradycardia

A

Correct Answer: C. Increased heart rate and enhanced contractility

Rationale: A hyperdynamic cardiac state in hyperthyroidism refers to increased heart rate (tachycardia) and myocardial contractility, which leads to increased cardiac output.

How well did you know this?
1
Not at all
2
3
4
5
Perfectly
96
Q

What gastrointestinal symptom is frequently observed in hyperthyroidism?
A. Constipation
B. Frequent bowel movements or diarrhea
C. Gastroesophageal reflux
D. Peptic ulcer disease

A

Correct Answer: B. Frequent bowel movements or diarrhea

Rationale: Hyperthyroidism often causes an increase in gastrointestinal motility, resulting in symptoms such as frequent bowel movements or diarrhea.

How well did you know this?
1
Not at all
2
3
4
5
Perfectly
97
Q

What psychological symptom may be present in a patient with hyperthyroidism?
A. Lethargy
B. Emotional stability
C. Emotionally unstable
D. Depression

A

Correct Answer: C. Emotionally unstable

Rationale: Hyperthyroidism can cause various psychological symptoms, including emotional instability, anxiety, and mood swings.

How well did you know this?
1
Not at all
2
3
4
5
Perfectly
98
Q

Graves’ disease is most commonly diagnosed in which population?
A. Males, 50-60 years old
B. Females, 20-40 years old
C. Children under 15 years old
D. Postmenopausal women

A

Correct Answer: B. Females, 20-40 years old

Rationale: Graves’ disease typically occurs in females, with a female-to-male ratio of about 7:1, and most commonly presents in individuals between 20-40 years of age.

How well did you know this?
1
Not at all
2
3
4
5
Perfectly
99
Q

What is the characteristic antibody associated with the diagnosis of Graves’ disease?
A. Antinuclear antibody (ANA)
B. Rheumatoid factor (RF)
C. Thyroid-stimulating hormone receptor antibody (TSHR-Ab)
D. Anti-thyroid peroxidase antibody (anti-TPO)

A

Correct Answer: C. Thyroid-stimulating hormone receptor antibody (TSHR-Ab)

Rationale: Graves’ disease is characterized by the presence of thyroid-stimulating antibodies that bind to and activate the TSH receptors, leading to thyroid growth, increased vascularity, and hypersecretion of thyroid hormones.

How well did you know this?
1
Not at all
2
3
4
5
Perfectly
100
Q

Which symptom is a hallmark of ophthalmopathy associated with Graves’ disease?
A. Cataracts
B. Glaucoma
C. Exophthalmos
D. Macular degeneration

A

C. Exophthalmos

Rationale: Ophthalmopathy associated with Graves’ disease often manifests as exophthalmos, which is the protrusion of the eyeballs. This can occur in approximately 30% of cases and is due to inflammation and edema in the orbital tissues.

How well did you know this?
1
Not at all
2
3
4
5
Perfectly
101
Q

In severe cases, what life-threatening symptom can arise from the enlargement of the thyroid gland in Graves’ disease?
A. Apnea
B. Inspiratory stridor
C. Hemoptysis
D. Aphonia

A

Correct Answer: B. Inspiratory stridor

Rationale: Severe enlargement of the thyroid gland in Graves’ disease can lead to compression of the trachea, which may present with inspiratory stridor, a high-pitched wheezing sound indicative of airway obstruction.

also, dysphagia, globus sensation

102
Q

What is the first-line treatment for Graves’ disease?
A. Corticosteroids
B. methimazole or propylthiouracil (PTU)
C. Immediate thyroidectomy
D. High-dose iodine alone

A

Correct Answer: B. Antithyroid medications such as methimazole or propylthiouracil (PTU)

Rationale: The initial treatment for Graves’ disease typically involves antithyroid medications like methimazole or propylthiouracil (PTU), which help to reduce the production of thyroid hormones.

103
Q

How do beta-blockers provide symptomatic relief in Graves’ disease?
A. By stimulating the thyroid to produce less hormone
B. By inhibiting the synthesis of TSH in the pituitary gland
C. By impeding the peripheral conversion of T4 to T3
D. By directly reducing the size of the thyroid gland

A

Correct Answer: C. By impeding the peripheral conversion of T4 to T3

Rationale: Beta-blockers, such as propranolol, are used in the symptomatic management of Graves’ disease because they can block the peripheral conversion of T4 to the more active T3, and they provide symptomatic relief from adrenergic symptoms like tachycardia and palpitations.

104
Q

What is a potential complication of surgical treatment for Graves’ disease?
A. Diabetic ketoacidosis
B. Acute adrenal crisis
C. Hypoparathyroidism
D. Hyperthyroid storm

A

Correct Answer: C. Hypoparathyroidism

Rationale: Complications from thyroid surgery can include hypothyroidism, hemorrhage with tracheal compression, damage to the recurrent laryngeal nerve (RLN), and damage to or inadvertent removal of the parathyroid glands, which can lead to hypoparathyroidism.

105
Q

When is ablative therapy or surgery typically considered in the treatment of Graves’ disease?
A. As the first-line treatment before trying medications
B. When the patient requests a non-pharmaceutical approach
C. When medical treatment has failed to control the disease
D. Immediately after diagnosis to prevent disease progression

A

Correct Answer: C. When medical treatment has failed to control the disease

Rationale: Ablative therapy or surgery is recommended for patients with Graves’ disease when medical treatment with antithyroid drugs is not successful in managing the disease, or in cases where rapid control of hyperthyroidism is necessary, such as thyroid storm or in preparation for surgery.

106
Q

What is the recommended duration for antithyroid drugs to take effect before proceeding with elective surgery for Graves’ disease?
A. 1-2 weeks
B. 2-4 weeks
C. 6-8 weeks
D. 3-6 months

A

Correct Answer: C. 6-8 weeks

Rationale: For elective surgical cases in patients with Graves’ disease, it is generally recommended to wait 6-8 weeks to allow for antithyroid drugs to take effect and stabilize thyroid hormone levels.

107
Q

Why is preoperative evaluation of the upper airway crucial in patients with Graves’ disease?
A. To assess for risk of postoperative pneumonia
B. To determine the need for intraoperative oxygen supplementation
C. To evaluate for evidence of tracheal compression or deviation caused by a goiter
D. To screen for obstructive sleep apnea

A

Correct Answer: C. To evaluate for evidence of tracheal compression or deviation caused by a goiter

Rationale: The preoperative assessment of the upper airway in patients with Graves’ disease is important to identify any anatomical deviation or compression of the trachea by an enlarged thyroid gland (goiter), which could complicate airway management during surgery.

108
Q

In emergent cases of Graves’ disease requiring surgery, which treatments are commonly used to rapidly prepare the patient?
A. Oral antithyroid drugs and aspirin
B. IV beta-blockers, glucocorticoids, and propylthiouracil (PTU)
C. High-dose iodine and calcium channel blockers
D. Antibiotics and antihypertensive medications

A

Correct Answer: B. IV beta-blockers, glucocorticoids, and propylthiouracil (PTU)

Rationale: In emergent situations, such as thyroid storm or when urgent surgery is required, intravenous beta-blockers, glucocorticoids, and propylthiouracil (PTU) are administered to quickly control hyperthyroid symptoms and stabilize the patient.

109
Q

What is the purpose of establishing thyroid levels preoperatively in patients with Graves’ disease?
A. To confirm the diagnosis of Graves’ disease
B. To optimize the thyroid state and minimize the risk of thyroid storm during surgery
C. To choose the appropriate type of surgical anesthesia
D. To assess the potential for allergic reactions to anesthetic agents

A

Correct Answer: B. To optimize the thyroid state and minimize the risk of thyroid storm during surgery

Rationale: Establishing and optimizing thyroid hormone levels preoperatively in patients with Graves’ disease is critical to reduce the risk of perioperative complications, including the potentially life-threatening thyroid storm.

110
Q

What is a thyroid storm often precipitated by?
A. Hypothyroidism
B. Elective surgery
C. Trauma or infection
D. Overdose of thyroid hormone replacement

A

Correct Answer: C. Trauma or infection

Rationale: A thyroid storm is a life-threatening exacerbation of hyperthyroidism that can be precipitated by stressors such as trauma, infection, medical illness, or surgery, especially when the hyperthyroid state is not well-controlled.

111
Q

How can thyroid storm be differentiated from malignant hyperthermia, given their similar presentations?
A. Thyroid storm typically presents with hypothermia.
B. Malignant hyperthermia does not involve altered mental status.
C. Thyroid storm can be associated with a history of thyroid disease.
D. Malignant hyperthermia occurs without a trigger such as anesthesia.

A

Correct Answer: C. Thyroid storm can be associated with a history of thyroid disease.

Rationale: Differentiation between thyroid storm and malignant hyperthermia is challenging due to similar clinical presentations. However, a history of thyroid disease and specific triggers may help distinguish thyroid storm. Thyroid storm is associated with a history of hyperthyroidism and can occur spontaneously or be precipitated by a stressor, whereas malignant hyperthermia is typically triggered by specific anesthetic agents.

112
Q

What is a common postoperative complication in patients with untreated or inadequately treated hyperthyroidism?
A. Myxedema coma
B. Thyroid storm
C. Hashimoto’s encephalopathy
D. Adrenal crisis

A

Correct Answer: B. Thyroid storm

Rationale: Thyroid storm is a critical condition that can occur postoperatively, especially in patients with untreated or inadequately treated hyperthyroidism who undergo emergency surgery.

113
Q

What is the primary treatment goal in a patient experiencing a thyroid storm?
A. Long-term suppression of thyroid hormone synthesis
B. Rapid alleviation of thyrotoxicosis and supportive care
C. Immediate surgical removal of the thyroid gland
D. Slow tapering of thyroid hormone levels over several weeks

A

Correct Answer: B. Rapid alleviation of thyrotoxicosis and supportive care

Rationale: The treatment of thyroid storm focuses on the rapid alleviation of thyrotoxicosis through antithyroid medications, supportive care to stabilize vital functions, and management of the precipitating factors.

114
Q

What is the mortality rate associated with a thyroid storm?
A. 5%
B. 10%
C. 20%
D. 50%

A

Correct Answer: C. 20%

Rationale: Despite aggressive treatment, the mortality rate for thyroid storm is high, historically around 20%, due to the severe systemic effects of this condition.

115
Q

What is the most common cause of primary hypothyroidism?
A. Autoimmune disease
B. Iodine deficiency
C. Ablation of the thyroid gland
D. Hereditary defects in thyroid hormone synthesis

A

Correct Answer: C. Ablation of the thyroid gland

Rationale: The most common cause of primary hypothyroidism in areas with sufficient iodine intake is ablation of the thyroid gland, typically due to treatment for hyperthyroidism with radioactive iodine or surgery.

116
Q

Which condition is associated with primary hypothyroidism and characterized by the presence of antibodies that block TSH receptors?
A. Graves’ disease
B. Hashimoto’s thyroiditis
C. Thyroid carcinoma
D. Idiopathic hypothyroidism

A

Correct Answer: D. Idiopathic hypothyroidism

Rationale: Idiopathic hypothyroidism, which is often thought to be autoimmune in nature, can be associated with the presence of antibodies that block TSH receptors, leading to decreased production of T3 and T4 despite normal or elevated TSH levels.

117
Q

Hashimoto’s thyroiditis primarily affects which demographic?
A. Elderly men
B. Middle-aged women
C. Young children
D. Adolescent males

A

Correct Answer: B. Middle-aged women

Rationale: Hashimoto’s thyroiditis is an autoimmune thyroid disorder more commonly diagnosed in middle-aged women and is characterized by a goitrous enlargement of the thyroid gland and hypothyroidism.

118
Q

What syndrome is commonly associated with hypothyroidism that includes hyponatremia and fluid overload?
A. Cushing’s syndrome
B. Addison’s disease
C. Syndrome of inappropriate antidiuretic hormone secretion (SIADH)
D. Polycystic ovary syndrome (PCOS)

A

Correct Answer: C. Syndrome of inappropriate antidiuretic hormone secretion (SIADH)

Rationale: SIADH, which can present with hyponatremia and fluid overload, is a common extrathyroidal manifestation of hypothyroidism.

119
Q

What neurologic symptom is characteristic of hypothyroidism?
A. Hyperreflexia
B. Slowing of motor function and prolonged relaxation phase of deep tendon reflexes
C. Increased intracranial pressure
D. Seizures

A

Correct Answer: B. Slowing of motor function and prolonged relaxation phase of deep tendon reflexes

Rationale: Neurologic manifestations of hypothyroidism include slow speech, general slowing of motor functions, and a prolonged relaxation phase following deep tendon reflexes, reflecting the overall slowing of metabolic processes in the body.

120
Q

Which cardiovascular finding is typically observed on electrocardiogram (ECG) in a patient with hypothyroidism?
A. Tall, peaked T waves
B. Flattened or inverted T waves
C. Prolonged QT interval
D. Wide QRS complexes

A

Correct Answer: B. Flattened or inverted T waves

Rationale: Hypothyroidism can affect the heart, often leading to specific changes on an ECG, such as flattened or inverted T waves, low-amplitude P waves, and sometimes bradycardia and ventricular dysrhythmias.

121
Q

What skin-related symptom is often present in hypothyroidism?
A. Rashes and urticaria
B. Warm, flushed skin
C. Pale, cool, dry thickened skin, nonpitting peripheral edema
D. Hyperpigmentation

A

Correct Answer: C. Pale, cool, dry thickened skin, nonpitting peripheral edema

Rationale: Hypothyroidism can cause a variety of skin changes, including pale, cool, and dry thickened skin, often accompanied by nonpitting peripheral edema, which is commonly referred to as myxedema.

122
Q

How is secondary hypothyroidism diagnosed?
A. Elevated levels of T3 and T4 with normal TSH
B. Normal levels of T3 and T4 with elevated TSH
C. Reduced levels of T3, T4, and TSH
D. Elevated levels of TSH with reduced levels of T3 and T4

A

Correct Answer: C. Reduced levels of T3, T4, and TSH

Rationale: Secondary hypothyroidism is characterized by low levels of T3 and T4, as well as TSH, indicating a pituitary or hypothalamic origin of the disorder rather than a primary thyroidal defect.

122
Q

What percentage of women over 60 are estimated to have subclinical hypothyroidism?
A. 5%
B. 10%
C. 20%
D. 30%

A

Correct Answer: C. 20%

Rationale: Approximately 20% of women over the age of 60 have subclinical hypothyroidism, a condition with an elevated TSH level but normal thyroid hormone levels, which can be associated with an increased risk of coronary artery disease if TSH is greater than 10 milliunits/L.

123
Q

In the context of a TRH stimulation test, what indicates a pituitary cause of hypothyroidism?
A. An elevated response of TSH to administered TRH
B. No response of TSH to administered TRH
C. A decrease in TSH after TRH administration
D. An immediate drop in T3 and T4 levels after TRH administration

A

Correct Answer: B. No response of TSH to administered TRH

Rationale: A TRH stimulation test can confirm a pituitary cause of hypothyroidism if there is no increase in TSH levels following the administration of TRH, which would normally stimulate the pituitary to release TSH.

124
Q

What is a distinguishing feature of euthyroid sick syndrome?
A. High T3 & T4 with low TSH level
B. Low T3 & T4 with normal TSH level
C. Normal T3 & T4 with high TSH level
D. High T3 with low T4 and TSH level

A

Correct Answer: B. Low T3 & T4 with normal TSH level

Rationale: Euthyroid sick syndrome, also known as nonthyroidal illness syndrome, is characterized by abnormal thyroid function tests, like low T3 and T4, in the absence of intrinsic thyroid disease, typically with a normal TSH level. It is seen in critically ill patients and is often a response to acute stress such as surgery, illness, or fasting.

125
Q

Which medication is considered the drug of choice (DOC) for the treatment of hypothyroidism and is crucial for perioperative management?

A) Methimazole
B) Propylthiouracil (PTU)
C) L-Thyroxine
D) Iodine Solution

A

Correct Answer: C) L-Thyroxine

Rationale: L-Thyroxine is the synthetic form of the thyroid hormone thyroxine and is the treatment of choice for hypothyroidism. It is essential in perioperative management to ensure euthyroid state and minimize the risk of myxedema coma, which can be precipitated by surgery and anesthesia.

126
Q

What is an important consideration for anesthesiologists regarding the gastrointestinal effects of hypothyroidism in the preoperative evaluation?

A) Increased risk of gastroesophageal reflux disease (GERD)
B) Slower gastric emptying increasing risk of aspiration
C) Increased bowel motility leading to diarrhea
D) Risk of gastric ulcer formation due to increased acidity

A

Correct Answer: B) Slower gastric emptying increasing risk of aspiration

Rationale: Hypothyroidism can lead to decreased gastrointestinal motility, resulting in slower gastric emptying. This condition increases the risk of aspiration during anesthesia induction. Preoperative fasting guidelines may need to be adjusted, and the use of prokinetic agents may be considered.

127
Q

In the context of hypothyroidism, which cardiovascular finding is an anesthesiologist likely to anticipate?

A) Hypertensive crisis
B) Hypodynamic cardiovascular system
C) High-output cardiac failure
D) Paroxysmal atrial tachycardia

A

Correct Answer: B) Hypodynamic cardiovascular system

Rationale: Patients with hypothyroidism often present with a hypodynamic cardiovascular system, characterized by decreased heart rate and cardiac output, which can affect hemodynamic management during anesthesia.

128
Q

Which strategy should be employed for thyroid hormone replacement if an elective surgery is planned for a hypothyroid patient?

A) Postpone surgery until a euthyroid state is achieved
B) Initiate L-thyroxine replacement intraoperatively
C) Start IV thyroid hormone replacement a week before surgery
D) Begin L-thyroxine therapy at least 10 days before surgery

A

Correct Answer: D) Begin L-thyroxine therapy at least 10 days before surgery

Rationale: It is recommended to start L-thyroxine at least 10 days before elective surgery to achieve a euthyroid state and reduce the risks associated with hypothyroidism during surgery. This timeframe allows for adequate stabilization of thyroid hormone levels.

129
Q

Which of the following is a complication that anesthesiologists must be vigilant about in patients with hypothyroidism undergoing surgery?

A) Hyperthermia
B) Electrolyte imbalances
C) Hyperglycemia
D) Hypocoagulability

A

Correct Answer: B) Electrolyte imbalances

Rationale: Hypothyroid patients are more prone to electrolyte imbalances, such as hyponatremia, which can be exacerbated by the stress of surgery and anesthesia. Careful monitoring and correction of electrolytes are vital to avoid perioperative complications.

If emergent surgery: IV Thyroid replacement along with steroids ASAP

130
Q

In managing a patient with myxedema coma, which intravenous medication is the initial treatment of choice?

A) IV Glucocorticoids
B) IV L-Thyroxine
C) IV Levothyroxine
D) IV Liothyronine

A

Correct Answer: B) IV L-Thyroxine

Rationale: Intravenous L-Thyroxine is the initial treatment of choice for myxedema coma. It can also be supplemented with L-Triiodothyronine in certain clinical situations given the urgent need for the restoration of thyroid hormone levels, considering the high mortality associated with myxedema coma.

131
Q

Which demographic is most commonly affected by myxedema coma?

A) Middle-aged men with recent thyroid surgery
B) Elderly women with a long history of hypothyroidism
C) Young women with newly diagnosed hypothyroidism
D) Children with congenital hypothyroidism

A

Correct Answer: B) Elderly women with a long history of hypothyroidism

Rationale: Myxedema coma is most commonly seen in elderly women with a long-standing history of hypothyroidism. This is due to physiological changes associated with aging and a potentially decreased ability to adapt to the hypothyroid state.

132
Q

What is a critical intervention required in the management of myxedema coma beyond thyroid hormone replacement?

A) Aggressive rewarming techniques
B) Correction of electrolyte imbalances
C) Immediate surgical intervention
D) High-dose antibiotic therapy

A

Correct Answer: B) Correction of electrolyte imbalances

Rationale: Electrolyte imbalances, particularly severe hyponatremia, are common in myxedema coma and must be carefully corrected along with other supportive measures such as mechanical ventilation if required, and stabilization of cardiac and pulmonary systems.

133
Q

Which clinical feature is a cardinal sign of myxedema coma?

A) Hyperthermia
B) Hypothermia
C) Diaphoresis
D) Tachycardia

A

Correct Answer: B) Hypothermia

Rationale: Hypothermia is a cardinal sign of myxedema coma due to impaired thermoregulation. Thyroid hormones play a significant role in thermogenesis and metabolism, and their deficiency leads to a decreased heat production.

134
Q

How quickly do vital signs typically improve after initiating treatment for myxedema coma?

A) Within 4 hours
B) Usually within 12 hours
C) Usually within 24 hours
D) Within 48-72 hours

A

Correct Answer: C) Usually within 24 hours

Rationale: Heart rate, blood pressure, and temperature typically show improvement within 24 hours after initiating appropriate treatment, which includes intravenous thyroid hormone replacement and supportive care. However, complete recovery may take longer and depends on the severity of the condition and promptness of the treatment.

135
Q

What is the primary medical treatment for most cases of goiter?

A) Radioactive iodine therapy
B) L-thyroxine supplementation
C) Beta-blockers
D) Antithyroid medications

A

Correct Answer: B) L-thyroxine supplementation

Rationale: Most goiters, especially those associated with a compensated euthyroid state, are treated with L-thyroxine supplementation. This treatment can help reduce the size of the goiter by inhibiting thyroid-stimulating hormone (TSH) secretion through the feedback mechanism on the pituitary gland.

136
Q

What is a common cause of goiter development due to the diet?

A) Excessive ingestion of iodine
B) Ingestion of goitrogenic substances
C) High consumption of selenium
D) Low intake of calcium

A

Correct Answer: B) Ingestion of goitrogenic substances

Rationale: Goiter can be caused by the ingestion of goitrogenic substances found in foods like cassava, or drugs such as phenylbutazone and lithium, which can interfere with thyroid hormone synthesis or release, leading to thyroid gland hypertrophy and hyperplasia.

137
Q

In which condition is surgery indicated for the treatment of goiter?

A) When there is a suspicion of malignancy
B) When the goiter is associated with hyperthyroidism
C) When medical therapy is ineffective and there are cosmetic or airway concerns
D) As a first-line treatment in all patients with a visible goiter

A

Correct Answer: C) When medical therapy is ineffective and there are cosmetic or airway concerns

Rationale: Surgical intervention for goiter is considered when medical therapy with L-thyroxine is ineffective and if the goiter compromises the airway (AW) or is cosmetically unacceptable to the patient.

138
Q

A goiter is typically associated with which of the following thyroid states?

A) Hypothyroidism
B) Hyperthyroidism
C) Euthyroidism
D) Thyroid storm

A

Correct Answer: C) Euthyroidism

Rationale: Although goiters can occur in various thyroid states, most cases of goiter are associated with a compensated euthyroid state, where the thyroid function tests are within normal ranges despite the gland enlargement.

139
Q

Which mechanism is primarily responsible for the swelling of the thyroid gland in goiter?

A) Inflammation of the thyroid gland
B) Hypertrophy and hyperplasia of follicular epithelium
C) Accumulation of thyroid hormone within the gland
D) Metastatic infiltration from other cancers

A

Correct Answer: B) Hypertrophy and hyperplasia of follicular epithelium

Rationale: The enlargement of the thyroid gland in goiter is due to hypertrophy and hyperplasia of the follicular epithelium. This is often a compensatory response to inadequate thyroid hormone production or increased TSH stimulation.

140
Q

What does a preoperative history of dyspnea in different positions (upright or supine) suggest in a patient with goiter?

A) Risk of airway (AW) obstruction during general anesthesia (GA)
B) Risk of postoperative pneumonia
C) Indication for bronchodilator therapy
D) Ineffective cough and risk for aspiration

A

Correct Answer: A) Risk of airway (AW) obstruction during general anesthesia (GA)

Rationale: Dyspnea in different body positions in a patient with goiter is suggestive of positional airway obstruction and is an important consideration in the preoperative evaluation, as it may predict airway compromise during anesthesia, necessitating careful airway management.

141
Q

Which diagnostic procedure is essential for evaluating the impact of a thyroid tumor on surrounding structures?

A) Thyroid function tests
B) CT scan
C) MRI of the neck
D) PET scan

A

Correct Answer: B) CT scan

Rationale: A CT scan of the neck and thorax is crucial for assessing the extent of a thyroid tumor, its anatomical relationship with surrounding structures, and planning for surgery or other treatments by providing detailed cross-sectional images.

142
Q

Flow-volume loops are used to:

A) Determine the function of the left ventricle
B) Assess the site and degree of airway obstruction
C) Measure the volume of pleural effusion
D) Evaluate the compliance of lung tissue

A

Correct Answer: B) Assess the site and degree of airway obstruction

Rationale: Flow-volume loops, performed in both upright and supine positions, are a pulmonary function test used to determine the site and degree of airway obstruction, which can be crucial in preoperative planning for patients with goiter.

143
Q

In flow-volume loop analysis, what does a limitation in the inspiratory limb of the loop indicate?

A) Intra-thoracic airway obstruction
B) Normal pulmonary function
C) Extra-thoracic airway obstruction
D) Restrictive lung disease

A

Correct Answer: C) Extra-thoracic airway obstruction

Rationale: Limitations in the inspiratory limb of the flow-volume loop suggest an extra-thoracic airway obstruction, which is consistent with conditions that compress or narrow the airway outside the thoracic cavity, such as a large goiter.

144
Q

An echocardiogram performed in both upright and supine positions can help evaluate:

A) The presence of valvular heart disease
B) The degree of cardiac compression by a goiter
C) Ejection fraction and cardiac output
D) Coronary artery patency

A

Correct Answer: B) The degree of cardiac compression by a goiter

Rationale: Echocardiography in different positions is useful in assessing the degree of cardiac compression, which can occur with large goiters, and helps to evaluate the impact of the thyroid mass on cardiac structures and function.

145
Q

What is the approximate morbidity rate associated with thyroid surgery?

A) 5%
B) 13%
C) 20%
D) 30%

A

Correct Answer: B) 13%

Rationale: Morbidity from thyroid surgery, which includes any complications arising from the procedure, approaches 13%. This encompasses a range of possible complications such as recurrent laryngeal nerve injury, hypoparathyroidism, and hematoma.

146
Q

Unilateral recurrent laryngeal nerve (RLN) injury post thyroid surgery typically presents with:

A) Stridor and risk of airway obstruction
B) Voice changes and hoarseness
C) Immediate need for tracheostomy
D) Permanent loss of vocal function

A

Correct Answer: B) Voice changes and hoarseness

Rationale: Unilateral RLN injury after thyroid surgery most commonly results in hoarseness due to impaired vocal cord movement. Complete airway obstruction is rare with unilateral involvement, and recovery of function generally occurs within 3-6 months if the nerve has not been transected.

147
Q

Postoperative hypocalcemia due to hypoparathyroidism typically occurs within what time frame after thyroid surgery?

A) Immediately postoperative
B) Within the first 6-12 hours postoperatively
C) In the first 24-48 hours postoperatively
D) 3-5 days postoperatively

A

Correct Answer: C) In the first 24-48 hours postoperatively

Rationale: Symptoms of hypocalcemia can manifest in the first 24-48 hours postoperatively due to inadvertent damage to the parathyroid glands during thyroid surgery. Prompt recognition and treatment of this complication are crucial to prevent severe symptoms.

148
Q

Which complication of thyroid surgery may require the immediate availability of a tracheostomy set at the bedside during the postoperative period?

A) Hypoparathyroidism
B) Thyroid storm
C) Hematoma causing tracheal compression
D) Unilateral RLN

A

C) Hematoma causing tracheal compression

149
Q

What is a potential consequence of bilateral RLN injury during thyroid surgery?

A) Improvement in voice quality
B) Temporary mild hoarseness
C) Airway obstruction and possible need for tracheostomy
D) No impact on vocal or respiratory function

A

Correct Answer: C) Airway obstruction and possible need for tracheostomy

Rationale: Bilateral RLN injury is a more serious complication than unilateral injury as it may cause airway obstruction and difficulty coughing, which can be life-threatening and may warrant a tracheostomy.

150
Q

Hypoparathyroidism following thyroid surgery is most likely due to:

A) Hyperplasia of the remaining parathyroid glands
B) Autoimmune destruction of parathyroid tissue
C) Inadvertent damage to the parathyroid glands
D) Metastatic involvement of the parathyroid glands

A

Correct Answer: C) Inadvertent damage to the parathyroid glands

Rationale: Hypoparathyroidism post-thyroidectomy is often the result of inadvertent damage to or removal of the parathyroid glands, leading to hypocalcemia, which typically presents in the first 24-48 hours after surgery.

151
Q

Postoperative management of a thyroidectomy patient should include:

A) Early removal of airway support
B) Immediate commencement of anti-thyroid medications
C) Observation and readiness to manage potential tracheal compression due to hematoma
D) Prophylactic use of antibiotics to prevent infection

A

Correct Answer: C) Observation and readiness to manage potential tracheal compression due to hematoma

Also, laryngospasm from hypocalcemia

Rationale: After thyroid surgery, there is a risk of hematoma formation that can compress the trachea. Hence, there should be vigilance and readiness to manage this, including keeping a tracheostomy set at the bedside during the immediate postoperative period.

152
Q

The adrenal cortex is responsible for the production of which of the following sets of hormones?

A) Glucocorticoids, mineralocorticoids, and catecholamines
B) Glucocorticoids, mineralocorticoids, and androgens
C) Catecholamines, glucocorticoids, and thyroid hormones
D) Mineralocorticoids, androgens, and insulin

A

Correct Answer: B) Glucocorticoids, mineralocorticoids, and androgens

Rationale: The adrenal cortex synthesizes and secretes glucocorticoids (e.g., cortisol), mineralocorticoids (e.g., aldosterone), and androgens. The adrenal medulla, in contrast, is responsible for the production of catecholamines such as epinephrine and norepinephrine.

153
Q

Which hormone acts as a trigger for the adrenal cortex to produce cortisol?

A) Thyroid-stimulating hormone (TSH)
B) Adrenocorticotropic hormone (ACTH)
C) Luteinizing hormone (LH)
D) Insulin

A

Correct Answer: B) Adrenocorticotropic hormone (ACTH)

Rationale: ACTH, released from the anterior pituitary gland, stimulates the adrenal cortex to produce cortisol. This is part of the hypothalamic-pituitary-adrenal (HPA) axis.

154
Q

What is the effect of cortisol on blood glucose levels?

A) Decreases blood glucose by increasing cellular uptake
B) Increases blood glucose by stimulating gluconeogenesis
C) Has no effect on blood glucose levels
D) Decreases blood glucose by stimulating glycogen synthesis

A

Correct Answer: B) Increases blood glucose by stimulating gluconeogenesis

Rationale: Cortisol induces hyperglycemia, reflecting its role in gluconeogenesis (the formation of glucose from non-carbohydrate sources) and the inhibition of glucose uptake by cells, which helps to provide energy in response to stress.

155
Q

Cortisol and aldosterone collectively have what impact on electrolyte balance?

A) They cause sodium retention and potassium excretion.
B) They lead to sodium excretion and potassium retention.
C) They have no significant impact on electrolytes.
D) They lead to calcium excretion and magnesium retention.

A

Correct Answer: A) They cause sodium retention and potassium excretion.

Rationale: Cortisol has some mineralocorticoid activity and can contribute to sodium retention, while aldosterone specifically promotes sodium retention and potassium excretion in the kidneys, affecting fluid and electrolyte balance.

156
Q

Where within the adrenal gland is norepinephrine (NE) converted to epinephrine (EPI)?

A) In the adrenal cortex
B) In the adrenal medulla
C) In the zona glomerulosa
D) In the zona reticularis

A

Correct Answer: B) In the adrenal medulla

Rationale: The adrenal medulla is responsible for the conversion of norepinephrine to epinephrine. This conversion is facilitated by cortisol, which is produced in the adrenal cortex.

157
Q

Pheochromocytomas primarily arise from which cells within the sympathoadrenal system?

A) Adrenal cortex cells
B) Pancreatic islet cells
C) Pituitary gland cells
D) Chromaffin cells

A

Correct Answer: D) Chromaffin cells

Rationale: Pheochromocytomas are tumors that secrete catecholamines and arise from chromaffin cells of the sympathoadrenal system, specifically located within the adrenal medulla.

158
Q

In patients with pheochromocytoma, which serious cardiovascular conditions can uncontrolled catecholamine release precipitate?

A) Bradycardia, hypotension, and thrombosis
B) Hypertension, cerebrovascular accident (CVA), and myocardial infarction (MI)
C) Arrhythmias, cardiomyopathy, and pericarditis
D) Heart failure, aneurysm, and peripheral artery disease

A

Correct Answer: B) Hypertension, cerebrovascular accident (CVA), and myocardial infarction (MI)

Rationale: Excessive catecholamine release from pheochromocytomas can lead to malignant hypertension, which increases the risk of acute cardiovascular events such as stroke (CVA) and myocardial infarction (MI).

159
Q

What is the typical ratio of norepinephrine (NE) to epinephrine (EPI) secreted by most pheochromocytomas?

A) 85:15
B) 50:50
C) 15:85
D) 70:30

A

Correct Answer: A) 85:15

Rationale: Most pheochromocytomas secrete catecholamines with a predominance of norepinephrine (NE) over epinephrine (EPI) at a ratio of approximately 85:15, which is the inverse of the normal adrenal secretion where epinephrine is more predominant.

160
Q

What percentage of pheochromocytomas are located in the organ of Zuckerkandle?

A) 2%
B) 10%
C) 18%
D) 25%

A

Correct Answer: C) 18%

Rationale: Approximately 18% of pheochromocytomas occur in the organ of Zuckerkandle, which is a collection of chromaffin cells located at the abdominal aorta bifurcation. The majority, however, are found in the adrenal medulla.

161
Q

What is the hallmark diagnostic test for pheochromocytoma?

A) Serum thyroid hormone levels
B) 24-hour urine collection for metanephrines and catecholamines
C) Liver function tests
D) Serum cortisol levels

A

Correct Answer: B) 24-hour urine collection for metanephrines and catecholamines

Rationale: A 24-hour urine collection for metanephrines and catecholamines is a key diagnostic test for pheochromocytoma, as it measures the byproducts of catecholamine metabolism, which are typically elevated in this condition.

162
Q

Which imaging study is NOT typically used to localize a pheochromocytoma?

A) CT scan
B) MRI
C) I-metaiodobenzylguanidine (MIBG) scintigraphy
D) Ultrasound

A

Correct Answer: D) Ultrasound

Rationale: While ultrasound can be used to image the adrenal glands, CT scans and MRI are preferred for their superior detail. I-metaiodobenzylguanidine (MIBG) scintigraphy is a specific type of nuclear medicine scan used to localize catecholamine-secreting tumors like pheochromocytoma.

163
Q

The symptomatology of pheochromocytoma commonly includes:

A) Polyuria and polydipsia
B) Hypotension and weight gain
C) Headache, sweating, and orthostatic hypotension
D) Tachycardia and hyperthermia

A

Correct Answer: C) Headache, sweating, and orthostatic hypotension

Rationale: Classic symptoms of pheochromocytoma are often summarized as the ‘5 Ps’: pressure (hypertension), pain (headache), perspiration (sweating), palpitations, and pallor, including orthostatic hypotension due to volume depletion.

164
Q

Cardiac complications of pheochromocytoma may include all of the following EXCEPT:

A) Coronary vasoconstriction
B) Cardiomyopathy
C) Congestive heart failure (CHF)
D) Mitral valve prolapse

A

Correct Answer: D) Mitral valve prolapse

Rationale: While pheochromocytoma can lead to serious cardiac issues such as coronary vasoconstriction, cardiomyopathy, and congestive heart failure due to the effects of excessive catecholamines, mitral valve prolapse is not typically associated with it.

165
Q

The frequency and duration of pheochromocytoma attacks are best described as:

A) Consistent and brief, lasting less than 30 seconds
B) Infrequent and prolonged, lasting several days
C) Occasional to frequent, ranging from 1 minute to several hours
D) Continuous without fluctuations

A

Correct Answer: C) Occasional to frequent, ranging from 1 minute to several hours

Rationale: Attacks of pheochromocytoma can range from occasional to frequent and may last from as little as one minute to several hours. They may occur spontaneously or be triggered by factors such as injury, stress, or medications.

166
Q

What is the primary reason for using an α blocker preoperatively in patients with pheochromocytoma?

A) To increase intravascular volume
B) To lower blood pressure and decrease intravascular volume
C) To stimulate adrenergic receptors
D) To increase myocardial dysfunction

A

Correct Answer: B) To lower blood pressure and decrease intravascular volume

Rationale: The preoperative use of an α blocker in pheochromocytoma patients is to lower blood pressure and decrease intravascular volume, which helps prevent intraoperative hypertensive episodes and decreases the risk of myocardial dysfunction due to excessive catecholamine stimulation

167
Q

Phenoxybenzamine is characterized by which of the following properties?

A) Competitive α1 antagonist with no effect on α2 receptors
B) Noncompetitive α1 antagonist with some α2-blocking properties
C) Selective α2 antagonist with no effect on α1 receptors
D) Competitive β blocker without intrinsic sympathomimetic activity

A

Correct Answer: B) Noncompetitive α1 antagonist with some α2-blocking properties

Rationale: Phenoxybenzamine is a noncompetitive α1 antagonist that irreversibly binds to the receptor and also possesses some α2-blocking properties. This makes it a long-acting agent suitable for preoperative management of pheochromocytoma.

168
Q

What is the appropriate management for tachycardia that occurs after α blockade in a patient with pheochromocytoma?

A) Administer a calcium channel blocker (CCB)
B) Increase fluid administration
C) Treat with a beta-blocker (BB)
D) Give a direct-acting vasodilator

A

Correct Answer: C) Treat with a beta-blocker (BB)

Rationale: If tachycardia occurs after α blockade, it is appropriate to treat it with a beta-blocker. However, beta-blockers should never be used before adequate α blockade to avoid unopposed α agonism and subsequent hypertensive crisis.

169
Q

Why should a nonselective beta-blocker never be given before an α blocker in the management of pheochromocytoma?

A) It causes reflex tachycardia
B) It leads to unopposed α agonism, causing vasoconstriction and hypertensive crises
C) It results in a sudden increase in tumor size
D) It enhances the effects of catecholamines

A

Correct Answer: B) It leads to unopposed α agonism, causing vasoconstriction and hypertensive crises

Rationale: Administering a nonselective beta-blocker before α blockade can lead to unopposed α agonism because of the loss of β2-mediated vasodilation. This can result in severe vasoconstriction and hypertensive crises, which are dangerous in patients with pheochromocytoma.

170
Q

Calcium channel blockers (CCBs) are used in the management of pheochromocytoma for their ability to:

A) Trigger catecholamine release from the tumor
B) Control hypertension
C) Increase calcium entry into myocardial cells
D) Treat catecholamine-mediated cardiomyopathy

A

Correct Answer: B) Control hypertension

Rationale: CCBs are used in the management of pheochromocytoma primarily to control hypertension. They work by inhibiting the entry of calcium into cells, which reduces the force of contraction of the heart and dilates the arteries. Excessive calcium can trigger catecholamine release from the tumor and contribute to cardiomyopathy, which CCBs help to prevent.

171
Q

What differentiates ACTH-independent Cushing’s syndrome from ACTH-dependent Cushing’s syndrome?

A) ACTH-independent Cushing’s syndrome is characterized by high plasma ACTH levels.
B) ACTH-dependent Cushing’s syndrome results from abnormal adrenocortical tissue.
C) ACTH-independent Cushing’s syndrome has suppressed CRH and ACTH levels due to autonomous cortisol secretion.
D) ACTH-dependent Cushing’s syndrome is typically caused by benign or malignant adrenocortical tumors.

A

Correct Answer: C) ACTH-independent Cushing’s syndrome has suppressed CRH and ACTH levels due to autonomous cortisol secretion.

Rationale: ACTH-independent Cushing’s syndrome is characterized by excessive cortisol production by abnormal adrenocortical tissue that is not regulated by corticotropin-releasing hormone (CRH) and adrenocorticotropic hormone (ACTH), resulting in suppressed levels of these hormones.

172
Q

Acute ectopic ACTH (adrenocorticotropic hormone) syndrome, a form of ACTH-dependent Cushing’s syndrome, is most commonly associated with which type of cancer?

A) Breast carcinoma
B) Small cell lung carcinoma
C) Prostate carcinoma
D) Hepatocellular carcinoma

A

Correct Answer: B) Small cell lung carcinoma

Rationale: Acute ectopic ACTH syndrome is most often associated with small cell lung carcinoma, where the cancer cells produce ACTH, leading to ACTH-dependent Cushing’s syndrome.

173
Q

What is the most common cause of ACTH-independent Cushing’s syndrome?

A) Pituitary adenoma
B) Adrenal hyperplasia
C) Adrenocortical tumors
D) Exogenous corticosteroid administration

A

Correct Answer: C) Adrenocortical tumors

Rationale: Benign or malignant adrenocortical tumors are the most common cause of ACTH-independent Cushing’s syndrome. These tumors produce cortisol independently of ACTH regulation.

174
Q

What effect does a high plasma ACTH have in ACTH-dependent Cushing’s syndrome?

A) It decreases cortisol production.
B) It stimulates the adrenal cortex to produce excessive cortisol.
C) It suppresses cortisol production to normal levels.
D) It has no effect on cortisol production.

A

Correct Answer: B) It stimulates the adrenal cortex to produce excessive cortisol.

Rationale: In ACTH-dependent Cushing’s syndrome, high levels of plasma ACTH stimulate the adrenal cortex to produce excessive cortisol, leading to the clinical manifestations of Cushing’s syndrome.

175
Q

In the context of Cushing’s syndrome, an ACTH stimulation test would likely show which result in ACTH-independent disease?

A) Elevated ACTH response
B) Normal ACTH response
C) Suppressed ACTH response
D) No change in cortisol levels

A

Correct Answer: C) Suppressed ACTH response

Rationale: In ACTH-independent Cushing’s syndrome, because the source of excess cortisol is autonomous adrenal cortical tissue, CRH and ACTH levels are typically suppressed. An ACTH stimulation test would not increase cortisol levels further in this scenario.

176
Q

What diagnostic test is used to confirm the diagnosis of Cushing’s syndrome by measuring cortisol levels?

A) Serum ACTH test
B) Low-dose dexamethasone suppression test
C) 24-hour urine cortisol test
D) Salivary cortisol test

A

Correct Answer: C) 24-hour urine cortisol test

Rationale: The 24-hour urine cortisol test is used to confirm the diagnosis of Cushing’s syndrome as it measures the total cortisol excreted in the urine over a 24-hour period, which is elevated in Cushing’s syndrome.

177
Q

What is a distinguishing feature of Cushing’s syndrome related to cortisol production?

A) High plasma ACTH and high cortisol levels
B) Low plasma ACTH and low cortisol levels
C) High plasma ACTH with no change in cortisol levels
D) Low plasma ACTH and high cortisol levels

A

Correct Answer: D) Low plasma ACTH and high cortisol levels

Rationale: In Cushing’s syndrome, cortisol production can be elevated independent of plasma ACTH levels. In ACTH-independent Cushing’s syndrome, cortisol levels are high despite low or suppressed ACTH levels, due to autonomous adrenal cortisol production.

178
Q

What role does the high-dose dexamethasone suppression test serve in the evaluation of Cushing’s syndrome?

A) It is used to determine the effectiveness of cortisol replacement therapy.
B) It helps to distinguish between Cushing’s disease and ectopic ACTH syndrome.
C) It confirms the diagnosis of primary adrenal insufficiency.
D) It evaluates the potential for adrenocortical carcinoma.

A

Correct Answer: B) It helps to distinguish between Cushing’s disease and ectopic ACTH syndrome.

Rationale: The high-dose dexamethasone suppression test is used to differentiate between Cushing’s disease (pituitary-dependent Cushing’s syndrome) and ectopic ACTH syndrome. Cushing’s disease will typically show a response to high-dose dexamethasone with a decrease in cortisol production, whereas ectopic ACTH production (e.g., from a lung tumor) usually will not be suppressed by dexamethasone.

179
Q

Which of the following symptoms is NOT typically associated with Cushing’s syndrome?

A) Exophthalmos
B) Moon face
C) Muscle wasting
D) Glucose intolerance

A

Correct Answer: A) Exophthalmos

Rationale: Exophthalmos, or protruding eyes, is not a typical feature of Cushing’s syndrome. The classic symptoms of Cushing’s syndrome include central obesity with a rounded “moon face,” muscle wasting, thinning of the skin with easy bruising (ecchymoses), hypertension, glucose intolerance, and neuropsychiatric disturbances like depression and insomnia.

180
Q

Which assay is used to measure plasma ACTH levels when differentiating between ACTH-dependent and independent forms of Cushing’s syndrome?

A) Enzyme-linked immunosorbent assay (ELISA)
B) Radioimmunoassay
C) Immunoradiometric assay
D) Western blot assay

A

Correct Answer: C) Immunoradiometric assay

Rationale: Reliable measurements of plasma ACTH using immunoradiometric assays are essential when determining whether Cushing’s syndrome is ACTH dependent or independent. This assay is particularly sensitive and specific for ACTH, helping to discern the source of hypercortisolism.

181
Q

What is the primary surgical treatment for a pituitary microadenoma causing Cushing’s syndrome?

A) Craniotomy
B) Transsphenoidal microadenomectomy
C) Transcranial adenomectomy
D) Endoscopic pituitary surgery

A

Correct Answer: B) Transsphenoidal microadenomectomy

Rationale: Transsphenoidal microadenomectomy is the treatment of choice for resectable pituitary microadenomas causing Cushing’s syndrome. This approach is minimally invasive and allows for the removal of the tumor through the sphenoid sinus, minimizing brain tissue disruption.

182
Q

When is a bilateral total adrenalectomy indicated in the treatment of Cushing’s syndrome?

A) When there is a pituitary microadenoma
B) When there is an adrenal adenoma
C) When pituitary irradiation fails or is not an option
D) As the first line of treatment for all forms of Cushing’s syndrome

A

Correct Answer: C) When pituitary irradiation fails or is not an option

Rationale: Bilateral total adrenalectomy may be necessary when other treatments, such as pituitary irradiation, fail or are not suitable. This is often the case in ACTH-dependent Cushing’s syndrome when the source cannot be controlled, resulting in continued overproduction of cortisol by the adrenal glands.

183
Q

What must be carefully evaluated and treated preoperatively in a patient with Cushing’s syndrome?

A) Bone density and calcium levels
B) Blood pressure, electrolyte balance, and blood glucose
C) Renal function and proteinuria
D) Liver function tests and bilirubin levels

A

Correct Answer: B) Blood pressure, electrolyte balance, and blood glucose

Rationale: Preoperative evaluation for a patient with Cushing’s syndrome must include careful assessment and stabilization of blood pressure, electrolyte imbalances (especially hypokalemia), and elevated blood glucose levels, as these can significantly impact perioperative risk and anesthetic management.

184
Q

What consideration should be given to positioning a patient with Cushing’s syndrome during surgery?

A) Risk for aspiration due to obesity
B) Risk for thermal injury due to skin thinning
C) Risk for pathologic fractures due to osteoporosis
D) Risk for pressure sores due to muscle wasting

A

Correct Answer: C) Risk for pathologic fractures due to osteoporosis

Rationale: When positioning a patient with Cushing’s syndrome for surgery, it’s important to consider the increased risk for pathologic fractures due to osteoporosis. Careful positioning and padding are necessary to prevent fractures in these patients who may have fragile bones.

185
Q

Which treatment is indicated for Cushing’s syndrome caused by an adrenal adenoma or carcinoma?

A) Pituitary irradiation
B) Pharmacological therapy with ketoconazole
C) Surgical adrenalectomy
D) Medical management with metyrapone

A

Correct Answer: C) Surgical adrenalectomy

Rationale: Surgical removal of the adrenal gland(s), or adrenalectomy, is indicated for Cushing’s syndrome caused by adrenal adenomas or carcinomas to remove the source of excessive cortisol production.

186
Q

Primary hyperaldosteronism is most commonly due to:

A) Renal artery stenosis
B) A functional adrenal tumor
C) Heart failure
D) Liver cirrhosis

A

Correct Answer: B) A functional adrenal tumor

Rationale: Primary hyperaldosteronism, also known as Conn’s syndrome, is most commonly caused by a functional adrenal tumor (aldosteronoma) that secretes aldosterone independently of the usual regulatory mechanisms.

187
Q

Which condition is NOT commonly associated with primary hyperaldosteronism?

A) Pheochromocytoma
B) Hyperparathyroidism
C) Acromegaly
D) Hypothyroidism

A

Correct Answer: D) Hypothyroidism

Rationale: Primary hyperaldosteronism is occasionally associated with other endocrine disorders, such as pheochromocytoma, hyperparathyroidism, and acromegaly. Hypothyroidism is not commonly associated with it.

188
Q

What triggers the release of aldosterone in secondary hyperaldosteronism?

A) Decreased serum potassium
B) Increased serum renin
C) Decreased serum sodium
D) Increased serum ACTH

A

Correct Answer: B) Increased serum renin

Rationale: Secondary hyperaldosteronism is characterized by the overproduction of aldosterone due to an external stimulus, most commonly increased serum renin, which can be due to conditions such as renal artery stenosis or congestive heart failure.

189
Q

Which of the following is a typical symptom of hyperaldosteronism?

A) Hypertension
B) Hyperkalemia
C) Metabolic acidosis
D) Hyponatremia

A

Correct Answer: A) Hypertension

Rationale: One of the main symptoms of hyperaldosteronism is hypertension, which is often accompanied by hypokalemia and metabolic alkalosis. This is due to the effect of aldosterone on the renal tubules, where it promotes sodium retention and potassium excretion.

190
Q

In the context of hyperaldosteronism, the metabolic disturbance commonly observed is:

A) Hypokalemic metabolic alkalosis
B) Hyperkalemic metabolic acidosis
C) Hypokalemic metabolic acidosis
D) Hyperkalemic metabolic alkalosis

A

Correct Answer: A) Hypokalemic metabolic alkalosis

Rationale: Hyperaldosteronism typically leads to hypokalemia and metabolic alkalosis. This results from the action of aldosterone increasing sodium reabsorption and potassium excretion in the kidneys, which also leads to the excretion of hydrogen ions, causing alkalosis.

191
Q

The presence of spontaneous hypokalemia in a patient with hypertension suggests which condition?

A) Primary hyperaldosteronism
B) Essential hypertension
C) Cushing’s syndrome
D) Pheochromocytoma

A

Correct Answer: A) Primary hyperaldosteronism

Rationale: Spontaneous hypokalemia in the presence of systemic hypertension is highly suggestive of hyperaldosteronism, particularly primary hyperaldosteronism, where there is excessive aldosterone production leading to increased renal potassium loss and sodium retention, causing hypertension.

192
Q

What is the expected plasma renin activity in primary hyperaldosteronism?

A) Elevated
B) Suppressed
C) Unchanged
D) Erratically variable

A

Correct Answer: B) Suppressed

Rationale: In primary hyperaldosteronism, plasma renin activity is typically suppressed due to the negative feedback mechanism; the high aldosterone level causes sodium retention and hypertension, which decreases the release of renin.

193
Q

Which substance, when ingested long-term, can cause a syndrome that mimics the features of hyperaldosteronism?

A) Licorice
B) Caffeine
C) Alcohol
D) Aspirin

A

Correct Answer: A) Licorice

Rationale: Long-term ingestion of licorice can cause a syndrome that mimics the features of hyperaldosteronism. Licorice contains glycyrrhizic acid, which can inhibit the enzyme 11β-hydroxysteroid dehydrogenase type 2, resulting in features such as hypertension, hypokalemia, and suppression of the renin-angiotensin-aldosterone system (RAAS).

194
Q

In the treatment of hyperaldosteronism, which medication is used as a competitive aldosterone antagonist?

A) Lisinopril
B) Hydrochlorothiazide
C) Spironolactone
D) Amlodipine

A

Correct Answer: C) Spironolactone

Rationale: Spironolactone is a competitive aldosterone antagonist used in the treatment of hyperaldosteronism. It blocks the effects of aldosterone on the renal tubules, thereby mitigating the potassium loss and hypertension associated with this condition.

195
Q

What might be the definitive treatment for an aldosterone-producing tumor in the context of hyperaldosteronism?

A) Beta-blocker therapy
B) Adrenalectomy
C) Continuous positive airway pressure (CPAP) therapy
D) High fluid intake

A

Correct Answer: B) Adrenalectomy

Rationale: Surgical removal of the adrenal gland, or adrenalectomy, may be the definitive treatment for an aldosterone-producing tumor in hyperaldosteronism. This would eliminate the source of excessive aldosterone secretion and potentially cure the condition.

196
Q

Hyperkalemia without renal insufficiency is a hallmark of which condition?

A) Primary hyperparathyroidism
B) Diabetes insipidus
C) Hypoaldosteronism
D) SIADH (Syndrome of Inappropriate Antidiuretic Hormone)

A

Correct Answer: C) Hypoaldosteronism

Rationale: Hyperkalemia in the absence of renal insufficiency is suggestive of hypoaldosteronism, where there is inadequate production or secretion of aldosterone, leading to decreased excretion of potassium by the kidneys.

197
Q

Which condition can enhance hyperkalemia in hypoaldosteronism?

A) Hyponatremia
B) Hypoglycemia
C) Hyperglycemia
D) Hypercalcemia

A

Correct Answer: C) Hyperglycemia

Rationale: Hyperkalemia may be enhanced by hyperglycemia. High blood sugar can cause a shift of potassium out of cells into the bloodstream, exacerbating the hyperkalemia already present in hypoaldosteronism.

198
Q

What electrolyte disturbance is commonly seen in hypoaldosteronism?

A) Hypochloremic metabolic alkalosis
B) Hyperchloremic metabolic acidosis
C) Hypernatremia
D) Hypocalcemia

A

Correct Answer: B) Hyperchloremic metabolic acidosis

Rationale: Hypoaldosteronism often leads to hyperchloremic metabolic acidosis due to impaired renal acid secretion and a decrease in sodium reabsorption, which aldosterone normally promotes.

199
Q

A reversible cause of hypoaldosteronism related to medication use is:

A) Aspirin-induced nephropathy
B) Indomethacin-induced prostaglandin deficiency
C) Lithium-induced nephrogenic diabetes insipidus
D) Metformin-induced lactic acidosis

A

Correct Answer: B) Indomethacin-induced prostaglandin deficiency

Rationale: Indomethacin can cause reversible hypoaldosteronism due to its effect on prostaglandin synthesis. Prostaglandins can affect aldosterone secretion, and their deficiency may lead to reduced aldosterone levels.

200
Q

What is a typical treatment approach for hypoaldosteronism?

A) Restriction of sodium intake
B) Administration of thiazide diuretics
C) Liberal sodium intake and administration of fludrocortisone
D) Administration of calcium gluconate

A

Correct Answer: C) Liberal sodium intake and administration of fludrocortisone

Rationale: Treatment of hypoaldosteronism often includes liberal sodium intake to counteract the loss of sodium due to lack of aldosterone, and fludrocortisone, a synthetic mineralocorticoid, to replace the deficient aldosterone and promote sodium reabsorption and potassium excretion by the kidneys.

201
Q

Which type of adrenal insufficiency is characterized by a deficiency in both glucocorticoids and mineralocorticoids?

A) Primary adrenal insufficiency
B) Secondary adrenal insufficiency
C) Tertiary adrenal insufficiency
D) Isolated glucocorticoid deficiency

A

Correct Answer: A) Primary adrenal insufficiency

Rationale: Primary adrenal insufficiency, also known as Addison’s disease, occurs when the adrenal glands are damaged and unable to produce sufficient amounts of glucocorticoids, mineralocorticoids, and androgens.

202
Q

What is the most common cause of primary adrenal insufficiency?

A) Adrenal artery thrombosis
B) Tuberculosis
C) Autoimmune adrenalitis
D) Metastatic cancer

A

Correct Answer: C) Autoimmune adrenalitis

Rationale: The most common cause of primary adrenal insufficiency in developed countries is autoimmune destruction of the adrenal cortex, which is often referred to as autoimmune adrenalitis.

203
Q

In secondary adrenal insufficiency, which hormone’s production is typically not affected?

A) Cortisol
B) Aldosterone
C) Adrenaline
D) Testosterone

A

Correct Answer: B) Aldosterone

Rationale: Secondary adrenal insufficiency usually involves only a glucocorticoid deficiency because aldosterone secretion is primarily regulated by the renin-angiotensin system, not by ACTH.

204
Q

What is a characteristic clinical finding in primary adrenal insufficiency that is not typically seen in secondary adrenal insufficiency?

A) Hyperpigmentation
B) Hypertension
C) Obesity
D) Hyperkalemia

A

Correct Answer: A) Hyperpigmentation

Rationale: Hyperpigmentation is a characteristic finding in primary adrenal insufficiency due to elevated levels of ACTH, which also stimulates melanocytes. This is typically absent in secondary adrenal insufficiency, as ACTH levels are low or normal.

205
Q

What iatrogenic factor is most commonly associated with the development of secondary adrenal insufficiency?

A) Long-term use of synthetic glucocorticoids
B) Overuse of mineralocorticoid receptor antagonists
C) Excessive use of IV saline
D) Administration of recombinant ACTH

A

Correct Answer: A) Long-term use of synthetic glucocorticoids

Rationale: Long-term use of synthetic glucocorticoids is a common iatrogenic cause of secondary adrenal insufficiency. It can suppress the hypothalamic-pituitary axis, leading to reduced production of CRH and ACTH, and ultimately decreased cortisol production by the adrenal glands.

206
Q

What does a baseline cortisol level of <20 µg/dL after ACTH stimulation suggest?

A) Normal adrenal function
B) Hypercortisolism
C) Adrenal insufficiency
D) Ectopic ACTH production

A

Correct Answer: C) Adrenal insufficiency

Rationale: A baseline cortisol level of less than 20 µg/dL that remains low after ACTH stimulation indicates adrenal insufficiency. This shows that the adrenal cortex is not responding adequately to ACTH, suggesting an impairment of adrenal function.

207
Q

In the diagnosis of adrenal insufficiency, what does a “positive” ACTH stimulation test mean?

A) The adrenal glands respond appropriately to ACTH
B) The adrenal glands show a poor or no increase in cortisol production in response to ACTH
C) The patient exhibits signs of Cushing’s syndrome
D) The test was administered incorrectly

A

Correct Answer: B) The adrenal glands show a poor or no increase in cortisol production in response to ACTH

Rationale: A positive ACTH stimulation test in the context of adrenal insufficiency means that there is a poor response to ACTH, which is demonstrated by a lack of significant increase in cortisol levels after administration of synthetic ACTH.

208
Q

How is absolute adrenal insufficiency characterized?

A) High baseline cortisol level with an exaggerated response to ACTH
B) Low baseline cortisol level with no response to ACTH
C) Normal baseline cortisol level with a suppressed response to ACTH
D) High baseline cortisol level with a normal response to ACTH

A

Correct Answer: B) Low baseline cortisol level with no response to ACTH

Rationale: Absolute adrenal insufficiency is characterized by a low baseline cortisol level and a lack of response (or a minimal increase in cortisol levels) to the ACTH stimulation test. (positive)

209
Q

Relative adrenal insufficiency is indicated by which laboratory finding?

A) Low baseline cortisol level with a positive response to ACTH
B) High baseline cortisol level with a positive response to ACTH
C) Low baseline cortisol level with a negative response to ACTH
D) Higher baseline cortisol level with a positive ACTH stimulation test

A

Correct Answer: D) Higher baseline cortisol level with a positive ACTH stimulation test

Rationale: Relative adrenal insufficiency is indicated when the baseline cortisol level is higher compared to absolute adrenal insufficiency, but there is still a positive ACTH stimulation test indicating inadequate response of the adrenal glands to ACTH.

210
Q

What is the standard treatment for adrenal insufficiency?

A) ACTH
B) Thyroid hormone replacement
C) Steroids
D) Nonsteroidal anti-inflammatory drugs (NSAIDs)

A

Correct Answer: C) Steroids

Rationale: The treatment for adrenal insufficiency typically involves replacement of the deficient steroids, most commonly glucocorticoids such as hydrocortisone, and in the case of primary adrenal insufficiency, mineralocorticoids such as fludrocortisone are also replaced.

211
Q

Where are the parathyroid glands located in relation to the thyroid gland?

A) Within the thyroid tissue
B) In front of the upper and lower poles of the thyroid gland
C) Behind the upper and lower poles of the thyroid gland
D) At a distance from the thyroid gland, near the thymus

A

Correct Answer: C) Behind the upper and lower poles of the thyroid gland

Rationale: The four parathyroid glands are typically located behind the upper and lower poles of the thyroid gland, where they produce parathyroid hormone (PTH) in response to plasma calcium levels.

212
Q

What is the role of parathyroid hormone (PTH)?

A) Decrease plasma calcium levels
B) Maintain normal plasma calcium levels
C) Regulate phosphate levels only
D) Increase plasma magnesium levels

A

Correct Answer: B) Maintain normal plasma calcium levels

Rationale: Parathyroid hormone (PTH) is essential in maintaining normal plasma calcium levels. It does so by promoting the movement of calcium across the gastrointestinal tract, renal tubules, and bone.

213
Q

What stimulates the release of parathyroid hormone (PTH)?

A) Hypercalcemia
B) Hypocalcemia
C) Hyperphosphatemia
D) Hypophosphatemia

A

Correct Answer: B) Hypocalcemia

Rationale: Hypocalcemia stimulates the release of PTH as part of a negative feedback mechanism. PTH then acts to correct the low calcium levels by increasing calcium reabsorption and mobilization from bone.

214
Q

What effect does hypercalcemia have on parathyroid hormone (PTH) synthesis and release?

A) It stimulates PTH release.
B) It suppresses PTH synthesis and release.
C) It has no effect on PTH synthesis and release.
D) It increases the synthesis but decreases the release of PTH.

A

Correct Answer: B) It suppresses PTH synthesis and release.

Rationale: Hypercalcemia suppresses the synthesis and release of PTH, reducing its actions on calcium mobilization, absorption, and reabsorption, to help lower plasma calcium levels to normal.

215
Q

Which of the following is a consequence of parathyroid hormone (PTH) on the renal tubules?

A) It decreases calcium reabsorption.
B) It increases calcium excretion.
C) It increases calcium reabsorption.
D) It has no effect on calcium handling.

A

Correct Answer: C) It increases calcium reabsorption.

Rationale: PTH acts on the renal tubules to increase calcium reabsorption, which helps to elevate or maintain the plasma calcium level. PTH also increases the excretion of phosphate by the kidneys, preventing the formation of insoluble calcium phosphate, which would otherwise precipitate in tissues.

216
Q

What is the most common cause of primary hyperparathyroidism?

A) Benign parathyroid adenoma
B) Carcinoma
C) Parathyroid hyperplasia
D) Renal failure

A

Correct Answer: A) Benign parathyroid adenoma

Rationale: Benign parathyroid adenoma is the most common cause of primary hyperparathyroidism, accounting for approximately 90% of cases.

217
Q

Which symptom is NOT typically associated with hyperparathyroidism?

A) Sedation
B) Nausea and vomiting
C) Polyuria
D) Hypocalcemia

A

Correct Answer: D) Hypocalcemia

Rationale: Hyperparathyroidism is typically associated with hypercalcemia, not hypocalcemia. Symptoms often related to hyperparathyroidism include sedation, nausea/vomiting, reduced strength and sensation, polyuria, renal stones, peptic ulcer disease (PUD), and cardiac disturbances due to high calcium levels.

218
Q

Which diagnostic tests are used to confirm hyperparathyroidism?

A) Plasma PTH and TSH levels
B) Plasma calcium and 24-hour urinary calcium
C) Serum glucose and insulin levels
D) Serum creatinine and BUN levels

A

Correct Answer: B) Plasma calcium and 24-hour urinary calcium

Rationale: The diagnosis of hyperparathyroidism is confirmed by measuring plasma calcium and 24-hour urinary calcium levels. Elevated calcium with high or inappropriately normal PTH levels and increased urinary calcium excretion are indicative of the disease.

219
Q

What is the treatment of choice for primary hyperparathyroidism?

A) Chronic calcium supplementation
B) Surgical removal of abnormal parathyroid tissue
C) Administration of bisphosphonates
D) Lifelong vitamin D supplementation

A

Correct Answer: B) Surgical removal of abnormal parathyroid tissue

Rationale: The treatment of choice for primary hyperparathyroidism, especially when symptomatic or causing complications, is the surgical removal of the abnormal parathyroid tissue.

220
Q

What is secondary hyperparathyroidism a compensatory response to?

A) Primary hyperparathyroidism
B) Hypoparathyroidism
C) Hypocalcemia secondary to chronic renal failure (CRF)
D) Vitamin D overdose

A

Correct Answer: C) Hypocalcemia secondary to chronic renal failure (CRF)

Rationale: Secondary hyperparathyroidism is a compensatory response of the parathyroid glands to chronic hypocalcemia, which is commonly seen in conditions such as chronic renal failure (CRF). It is the parathyroid glands’ attempt to normalize calcium levels.

221
Q

Why does secondary hyperparathyroidism rarely produce hypercalcemia?

A) The condition is typically treated promptly before hypercalcemia can develop.
B) It results from a separate disease process that causes hypocalcemia, not from overproduction of PTH.
C) It only affects the renal handling of calcium, not its release from bones.
D) Patients with secondary hyperparathyroidism usually have hypervitaminosis D.

A

Correct Answer: B) It results from a separate disease process that causes hypocalcemia, not from overproduction of PTH.

Rationale: Secondary hyperparathyroidism is adaptive and serves to counteract hypocalcemia. It is due to a separate disease process that causes hypocalcemia, like chronic renal failure, so the increased PTH production is a response to this and usually does not produce hypercalcemia.

222
Q

What is a standard treatment approach for patients with secondary hyperparathyroidism and renal disease?

A) Administration of calcium supplements
B) Administering a phosphate binder
C) Increasing dietary potassium
D) Reducing dietary calcium intake

A

Correct Answer: B) Administering a phosphate binder

Rationale: Treatment for secondary hyperparathyroidism, particularly in the context of renal disease, includes normalizing phosphate levels by administering a phosphate binder. This helps reduce phosphate levels, which can contribute to hypocalcemia and thus drive secondary hyperparathyroidism.

223
Q

What condition is indicated by a low plasma calcium level (<4.5 mEq/L) and low PTH?

A) Hyperparathyroidism
B) Hypoparathyroidism
C) Secondary hyperparathyroidism
D) Vitamin D toxicity

A

Correct Answer: B) Hypoparathyroidism

Rationale: Hypoparathyroidism is characterized by low plasma calcium levels along with low levels of PTH, which is responsible for regulating calcium levels in the body. When PTH is deficient, calcium levels fall, leading to the symptoms associated with hypocalcemia.

224
Q

Which clinical feature is NOT commonly associated with chronic hypocalcemia?

A) Fatigue
B) Cataracts
C) Thickening of the skull
D) Hyperreflexia

A

Correct Answer: D) Hyperreflexia

Rationale: Chronic hypocalcemia is associated with symptoms such as fatigue, cramps, lethargy, cataracts, subcutaneous calcifications, thickening of the skull, and neurologic deficits. Hyperreflexia is typically not a feature of chronic hypocalcemia; it is more commonly associated with conditions that cause hypercalcemia or electrolyte imbalances affecting nerve excitability.

225
Q

What is pseudohypoparathyroidism?

A) A condition where PTH levels are high, but peripheral tissues are resistant to it
B) A congenital disorder where PTH is adequate, but kidneys are unable to respond to it
C) A condition where PTH is low because of dietary deficiencies
D) A condition where PTH is overproduced by lung tumors

A

Correct Answer: B) A congenital disorder where PTH is adequate, but kidneys are unable to respond to it

Rationale: Pseudohypoparathyroidism is a congenital disorder characterized by a normal or increased PTH level with end-organ resistance, particularly in the kidneys, resulting in features similar to hypoparathyroidism despite adequate levels of the hormone.

226
Q

What is the most common cause of chronic hypocalcemia?

A) Hypervitaminosis D
B) Chronic renal failure (CRF)
C) Hyperthyroidism
D) Magnesium deficiency

A

Correct Answer: B) Chronic renal failure (CRF)

Rationale: Chronic renal failure (CRF) is the most common cause of chronic hypocalcemia. In CRF, the kidneys are less able to convert vitamin D to its active form and excrete phosphate, leading to reduced calcium absorption and retention of phosphate, which further lowers calcium levels.

227
Q

What treatment is administered for hypoparathyroidism?

A) Phosphate binders
B) Corticosteroids
C) Calcium and Vitamin D replacement
D) Synthetic parathyroid hormone

A

Correct Answer: C) Calcium and Vitamin D replacement

Rationale: The mainstay of treatment for hypoparathyroidism is calcium and vitamin D replacement to correct hypocalcemia and maintain normal serum calcium levels. Vitamin D helps increase calcium absorption from the gut, and calcium supplementation helps directly increase serum calcium levels.

228
Q

What structure within the brain houses the pituitary gland?

A) Pineal gland
B) Sella turcica
C) Hypothalamus
D) Third ventricle

A

Correct Answer: B) Sella turcica

Rationale: The pituitary gland is located within a bony structure called the sella turcica at the base of the brain.

229
Q

Which hormones are synthesized in the hypothalamus but stored and released from the posterior pituitary?

A) ACTH and TSH
B) GH and prolactin
C) Vasopressin and oxytocin
D) FSH and LH

A

Correct Answer: C) Vasopressin and oxytocin

Rationale: Vasopressin (antidiuretic hormone) and oxytocin are synthesized in the hypothalamus and then transported down the pituitary stalk to be stored and released by the posterior pituitary gland.

230
Q

What triggers the release of hormones from the posterior pituitary gland?

A) Neural signals from the cerebral cortex
B) Feedback from peripheral hormone levels
C) Osmoreceptor sensing in the hypothalamus
D) Direct stimulation from circulating blood glucose

A

Correct Answer: C) Osmoreceptor sensing in the hypothalamus

Rationale: The stimulus for the release of hormones from the posterior pituitary gland, specifically vasopressin, arises from osmoreceptors in the hypothalamus that sense changes in plasma osmolarity.

231
Q

Overproduction of which hormone is commonly associated with Cushing’s syndrome?

A) Growth hormone (GH)
B) Thyroid-stimulating hormone (TSH)
C) Adrenocorticotropic hormone (ACTH)
D) Prolactin

A

Correct Answer: C) Adrenocorticotropic hormone (ACTH)

Rationale: Cushing’s syndrome is often associated with the hypersecretion of ACTH due to anterior pituitary adenomas. The excess ACTH stimulates the adrenal cortex to overproduce cortisol, leading to the clinical manifestations of Cushing’s syndrome.

232
Q

Which part of the pituitary gland is under direct control of the hypothalamus through vascular connections?

A) Anterior pituitary
B) Posterior pituitary
C) Both anterior and posterior pituitary
D) Neither, as the pituitary operates independently

A

Correct Answer: A) Anterior pituitary

Rationale: The anterior pituitary secretes hormones under the direct control of the hypothalamus through a specialized vascular system known as the hypophyseal portal system. This system allows hypothalamic hormones to regulate the activity of the anterior pituitary gland.

233
Q

What is a key diagnostic marker for acromegaly?

A) Elevated serum cortisol
B) Elevated serum calcium
C) Elevated serum insulin-like growth factor 1 (IGF-1)
D) Reduced serum thyroid hormone

A

Correct Answer: C) Elevated serum insulin-like growth factor 1 (IGF-1)

Rationale: Serum insulin-like growth factor 1 (IGF-1) is elevated in acromegaly and serves as a key diagnostic marker since it reflects the increased secretion of growth hormone (GH), typically due to a pituitary adenoma.

234
Q

What test can be used to confirm the diagnosis of acromegaly?

A) Thyroid function test
B) Oral glucose tolerance test (OGTT) with measurement of GH levels
C) Parathyroid hormone (PTH) assay
D) Adrenocorticotropic hormone (ACTH) stimulation test

A

Correct Answer: B) Oral glucose tolerance test (OGTT) with measurement of GH levels

Rationale: The oral glucose tolerance test (OGTT) measures plasma growth hormone levels after ingestion of 75g glucose. In healthy individuals, glucose ingestion suppresses GH levels, but in acromegaly, the GH level remains elevated above 1 ng/mL.

235
Q

Which symptom is commonly associated with acromegaly due to the overgrowth of soft tissues?

A) Upper airway (AW) obstruction
B) Hyperventilation
C) Hypoventilation
D) Bradycardia

A

Correct Answer: A) Upper airway (AW) obstruction

Rationale: The overgrowth of soft tissues in acromegaly can make patients more susceptible to upper airway obstruction due to the enlargement of the airway tissues and structures.

236
Q

What condition is commonly associated with acromegaly and can result from the overgrowth of surrounding cartilaginous structures?

A) Tinnitus
B) Epistaxis
C) Hoarseness and vocal cord abnormalities
D) Otitis media

A

Correct Answer: C) Hoarseness and vocal cord abnormalities

Rationale: Hoarseness and abnormal movement of vocal cords, or even recurrent laryngeal nerve (RLN) paralysis, may result from the overgrowth of the surrounding cartilaginous structures in patients with acromegaly.

237
Q

What is the first-line treatment for a pituitary adenoma causing acromegaly?

A) Radiotherapy
B) Chemotherapy
C) Transsphenoidal surgical excision of the adenoma
D) Hormone replacement therapy

A

Correct Answer: C) Transsphenoidal surgical excision of the adenoma

Rationale: The first-line treatment for a pituitary adenoma causing acromegaly is usually transsphenoidal surgical excision of the pituitary adenoma.

If surgery not feasible, txis a LA somatostatin analogue

238
Q

Why might patients with acromegaly require modified airway management during anesthesia?

A) The likelihood of vocal cord paralysis
B) An increased risk of aspiration due to esophageal dysmotility
C) Distorted facial anatomy and upper airway structures may complicate intubation
D) An increased risk of malignant hyperthermia

A

Correct Answer: C) Distorted facial anatomy and upper airway structures may complicate intubation

Rationale: Patients with acromegaly may have distorted facial anatomy due to overgrowth of facial bones and soft tissues, including an enlarged tongue and epiglottis. These changes can interfere with the placement of a face mask and visualization of vocal cords during direct laryngoscopy (DL), making airway management more challenging.

239
Q

What complication can arise from an enlarged tongue and epiglottis during anesthesia in a patient with acromegaly?

A) Decreased risk of airway obstruction
B) Improved visualization of vocal cords on direct laryngoscopy
C) Increased distance between the lips and vocal cords complicating intubation
D) Easier placement of a face mask

A

Correct Answer: C) Increased distance between the lips and vocal cords complicating intubation

Rationale: In acromegaly, an enlarged tongue and epiglottis can predispose to upper airway obstruction and make it difficult to visualize the vocal cords during direct laryngoscopy. The increased distance between the lips and vocal cords due to mandibular overgrowth can further complicate intubation.

240
Q

Which technique may be necessary for intubating a patient with acromegaly?

A) Routine intubation with a standard endotracheal tube (ETT)
B) Use of a smaller ETT and potential use of video laryngoscopy (VL) or awake fiberoptic intubation
C) Blind nasal intubation
D) Rigid bronchoscopy for intubation

A

Correct Answer: B) Use of a smaller ETT and potential use of video laryngoscopy (VL) or awake fiberoptic intubation

Rationale: Due to the potential for difficult airway management in patients with acromegaly, there may be a need for a smaller endotracheal tube (ETT) and advanced intubation techniques such as video laryngoscopy (VL) or awake fiberoptic intubation to secure the airway safely.

241
Q

How does vocal cord enlargement in acromegaly affect the glottic opening?

A) It widens the glottic opening
B) It causes no significant change to the glottic opening
C) It may narrow the glottic opening
D) It elongates the glottic opening vertically

A

Correct Answer: C) It may narrow the glottic opening

Rationale: Vocal cord enlargement, which can occur in acromegaly, may narrow the glottic opening. This can complicate intubation by making it difficult to pass the endotracheal tube through the vocal cords and necessitating the use of a smaller diameter tube.

242
Q

What is the primary defect in neurogenic diabetes insipidus (DI)?

A) Excessive production of vasopressin
B) Destruction of posterior pituitary or hypothalamus leading to vasopressin deficiency
C) Renal tubules are unresponsive to vasopressin
D) Overproduction of renin by the kidneys

A

Correct Answer: B) Destruction of posterior pituitary or hypothalamus leading to vasopressin deficiency

Rationale: Neurogenic DI reflects a deficiency in vasopressin (antidiuretic hormone, ADH) due to damage or destruction of the posterior pituitary gland or hypothalamus, resulting in the inability to concentrate urine.

243
Q

How is nephrogenic DI differentiated from neurogenic DI?

A) By the presence of high serum osmolarity
B) Through a water deprivation test
C) By response to desmopressin administration
D) By the measurement of urine osmolarity alone

A

Correct Answer: C) By response to desmopressin administration

Rationale: Neurogenic and nephrogenic DI are differentiated based on the response to desmopressin, a synthetic analogue of vasopressin. In neurogenic DI, desmopressin will decrease urine output and increase its osmolality, but in nephrogenic DI, the renal tubules are unresponsive to desmopressin, and there is little to no change in urine output or osmolality.

244
Q

What symptom is common to both neurogenic and nephrogenic DI?

A) Edema
B) Polydipsia and polyuria
C) Hypertension
D) Weight gain

A

Correct Answer: B) Polydipsia and polyuria

Rationale: Both forms of DI present with polydipsia (excessive thirst) and polyuria (high output of poorly concentrated urine), despite increased serum osmolarity, due to the body’s inability to conserve water without the effects of ADH.

245
Q

What is a key component of the initial treatment for diabetes insipidus?

A) Oral hypoglycemic agents
B) IV fluids to offset polyuria
C) Immediate use of diuretics
D) High-salt diet

A

Correct Answer: B) IV fluids (electrolytes) to offset polyuria

Rationale: The initial treatment for DI involves administration of IV fluids (electrolytes) to replace the large urine volume loss due to polyuria and to prevent dehydration.

246
Q

During anesthesia, what is essential to monitor in a patient with diabetes insipidus?

A) Urine output (UOP) and serum electrolyte concentrations
B) Only blood pressure
C) Only body temperature
D) Serum creatinine levels

A

Correct Answer: A) Urine output (UOP) and serum electrolyte concentrations

Rationale: During anesthesia, it is critical to monitor urine output and serum electrolyte concentrations in patients with DI, as they are prone to significant fluid and electrolyte imbalances due to their condition.

247
Q

What conditions can lead to the Syndrome of Inappropriate ADH secretion (SIADH)?

A) Diabetes mellitus and hypertension
B) Intracranial tumors and hypothyroidism
C) Pancreatitis and hypercalcemia
D) Adrenal insufficiency and hyperlipidemia

A

Correct Answer: B) Intracranial tumors and hypothyroidism

Rationale: SIADH can occur in the presence of various pathologies, including intracranial tumors, hypothyroidism, porphyria, and lung carcinoma, which can cause an inappropriate secretion of antidiuretic hormone (ADH).

248
Q

What is a hallmark sign of SIADH in laboratory findings?

A) Decreased serum sodium and increased serum osmolality
B) Increased urinary sodium and decreased serum osmolality
C) Decreased urinary sodium and decreased serum osmolality
D) Increased serum sodium and decreased urinary osmolality

A

Correct Answer: B) Increased urinary sodium and decreased serum osmolality

Rationale: In SIADH, the laboratory findings typically include inappropriately increased urinary sodium excretion and a decrease in serum osmolality in the presence of hyponatremia, which is not appropriate for the serum osmolality.

249
Q

What serious neurological complication can arise from a rapid decrease in serum sodium concentration in SIADH?

A) Peripheral neuropathy
B) Cerebral edema and seizures
C) Migraines
D) Transient ischemic attacks

A

Correct Answer: B) Cerebral edema and seizures

Rationale: Abrupt decreases in serum sodium concentration can cause an osmotic imbalance across the blood-brain barrier, leading to cerebral edema and potentially causing seizures.

250
Q

What medication is commonly used as an ADH antagonist in the treatment of SIADH?

A) Furosemide
B) Hydrochlorothiazide
C) Demeclocycline
D) Spironolactone

A

Correct Answer: C) Demeclocycline

Rationale: Demeclocycline is an ADH antagonist that is used in the treatment of SIADH. It reduces the action of ADH on the renal tubules, which can help correct the water retention and hyponatremia seen in SIADH.

251
Q

In the management of severe hyponatremia associated with SIADH, what is the recommended treatment?

A) Hypertonic saline
B) Oral hydration with water
C) Calcium gluconate infusion
D) Insulin and dextrose infusion

A

Correct Answer: A) Hypertonic saline

hypertonic saline @ <8 mEq/L over 24-hrs

Rationale: Severe hyponatremia in SIADH may be treated with hypertonic saline, especially when serum sodium is critically low (<120 mEq/L) and/or the patient is symptomatic. The infusion rate should be carefully controlled to avoid overly rapid correction, which can lead to osmotic demyelination syndrome.